Cardiac

Ace your homework & exams now with Quizwiz!

The nurse is teaching a group of teens about prevention of heart disease. Which point should the nurse emphasize? Reduce abdominal fat. Avoid stress. Do not smoke or chew tobacco. Avoid alcoholic beverages.

Do not smoke or chew tobacco. Tobacco exposure, including secondhand smoke, reduces coronary blood flow; causes vasoconstriction, endothelial dysfunction, and thickening of the vessel walls; increases carbon monoxide; and decreases oxygen. Because it is highly addicting, beginning smoking in the teen years may lead to decades of exposure. Teens are not likely to experience metabolic syndrome from obesity, but are very likely to use tobacco. Avoiding stress is a less modifiable risk factor, which is less likely to cause heart disease in teens. The risk of smoking outweighs the risk of alcohol use.

Which atypical symptoms may be present in a female client experiencing myocardial infarction (MI)? (Select all that apply.) Sharp, inspiratory chest pain Dyspnea Dizziness Extreme fatigue Anorexia

Dyspnea Dizziness Extreme fatigue Many women who experience an MI present with dyspnea, light-headedness, and fatigue. Sharp, pleuritic pain is more consistent with pericarditis or pulmonary embolism. Anorexia is neither a typical nor an atypical sign of MI.

After thrombolytic therapy, the nurse working in the cardiac catheterization laboratory would be alarmed to notice which sign? A 1-inch backup of blood in the IV tubing Facial drooping Partial thromboplastin time (PTT) 68 seconds Report of chest pressure during dye injection

Facial drooping During and after thrombolytic administration, the nurse observes for any indications of bleeding, including changes in neurologic status, which may indicate intracranial bleeding. A 1-inch backup of blood in the IV tubing may be related to IV positioning. If heparin is used, PTT reflects a therapeutic value. Reports of chest pressure during dye injection or stent deployment are considered an expected result of the procedure.

Which characteristics place women at high risk for myocardial infarction (MI)? (Select all that apply.) Premenopausal Increasing age Family history Abdominal obesity Breast cancer

Increasing age Family history Abdominal obesity Increasing age is a risk factor, especially after 70 years. Family history is a significant risk factor in both men and women. A large waist size and/or abdominal obesity are risk factors for both metabolic syndrome and MI. Premenopausal women are not at higher risk for MI, and breast cancer is not a risk factor for MI.

The visiting nurse is seeing a client postoperative for coronary artery bypass graft. Which nursing action should be performed first? Assess coping skills. Assess for postoperative pain at the client's incision site. Monitor for dysrhythmias. Monitor mental status.

Monitor for dysrhythmias. Dysrhythmias are the leading cause of prehospital death; the nurse should monitor the client's heart rhythm. Assessing mental status, coping skills, or postoperative pain is not the priority for this client.

When planning care for a client in the emergency department, which interventions are needed in the acute phase of myocardial infarction? (Select all that apply.) Morphine sulfate Oxygen Nitroglycerin Naloxone Acetaminophen Verapamil (Calan, Isoptin)

Morphine sulfate Oxygen Nitroglycerin Morphine is needed to reduce oxygen demand, preload, pain, and anxiety, and nitroglycerin is used to reduce preload and chest pain. Administering oxygen will increase available oxygen for the ischemic myocardium. Naloxone is a narcotic antagonist that is used for overdosage of opiates, not for MI. Acetaminophen may be used for headache related to nitroglycerin. Because of negative inotropic action, calcium channel blockers such as verapamil are used for angina, not for MI.

A client comes to the emergency department with chest discomfort. Which action does the nurse perform first? Administers oxygen therapy Obtains the client's description of the chest discomfort Provides pain relief medication Remains calm and stays with the client

Obtains the client's description of the chest discomfort A description of the chest discomfort must be obtained first, before further action can be taken. Neither oxygen therapy nor pain medication is the first priority in this situation; an assessment is needed first. Remaining calm and staying with the client are important, but are not matters of highest priority.

Prompt pain management with myocardial infarction is essential for which reason? The discomfort will increase client anxiety and reduce coping. Pain relief improves oxygen supply and decreases oxygen demand. Relief of pain indicates that the MI is resolving. Pain medication should not be used until a definitive diagnosis has been established.

Pain relief improves oxygen supply and decreases oxygen demand. The focus of pain relief is on reducing myocardial oxygen demand. Chest discomfort will increase anxiety, but it may not affect coping. Relief of pain is secondary to the use of opiates or indicates that the tissue infarction is complete. Although it used to be true that pain medication was not to be used for undiagnosed abdominal pain, this does not relate to MI.

The nurse is caring for a client in phase 1 cardiac rehabilitation. Which activity does the nurse suggest? The need to increase activities slowly at home Planning and participating in a walking program Placing a chair in the shower for independent hygiene Consultation with social worker for disability planning

Placing a chair in the shower for independent hygiene Phase 1 begins with the acute illness and ends with discharge from the hospital; it focuses on promoting rest and allowing clients to improve their activities of daily living based on their abilities. Phase 2 begins after discharge and continues through convalescence at home, including consultation with a social worker for long-term planning; it consists of achieving and maintaining a vital and productive life while remaining within the limits of the heart's ability to respond to increases in activity and stress. Phase 3 refers to long-term conditioning, such as a walking program.

The nurse is caring for a client 36 hours after coronary artery bypass grafting, with a priority problem of intolerance for activity related to imbalance of myocardial oxygen supply and demand. Which finding causes the nurse to terminate an activity and return the client to bed? Pulse 60 beats/min and regular Urinary frequency Incisional discomfort Respiratory rate 28 breaths/min

Respiratory rate 28 breaths/min Tachypnea and tachycardia reflect activity intolerance; activity should be terminated. Pulse 60 beats/min and regular is a normal finding. Urinary frequency may indicate infection or diuretic use, but not activity intolerance. Pain with activity after surgery is anticipated; pain medication should be available.

The nurse is assessing a client with chest pain to evaluate whether the client is suffering from angina or myocardial infarction (MI). Which symptom is indicative of an MI? Chest pain brought on by exertion or stress Substernal chest discomfort occurring at rest Substernal chest discomfort relieved by nitroglycerin or rest Substernal chest pressure relieved only by opioids

Substernal chest pressure relieved only by opioids Substernal chest pressure relieved only by opioids is typically indicative of MI. Substernal chest discomfort that occurs at rest is not necessarily indicative of MI; it could be a sign of unstable angina. Both chest pain brought on by exertion or stress and substernal chest discomfort relieved by nitroglycerin or rest are indicative of angina.

After receiving change-of-shift report in the coronary care unit, which client does the nurse assess first? The client with acute coronary syndrome who has a 3-pound weight gain and dyspnea The client with percutaneous coronary angioplasty who has a dose of heparin scheduled The client who had bradycardia after a myocardial infarction and now has a paced heart rate of 64 beats/min A client who has first-degree heart block, rate 68 beats/min, after having an inferior myocardial infarction

The client with acute coronary syndrome who has a 3-pound weight gain and dyspnea Dyspnea and weight gain are symptoms of left ventricular failure and pulmonary edema; this client needs prompt intervention. A scheduled heparin dose does not take priority over dyspnea; it can be administered after the client with dyspnea is taken care of. The client with a pacemaker and a normal heart rate is not in danger. First-degree heart block is rarely symptomatic, and the client has a normal heart rate; the client with dyspnea should be seen first.

The nurse is preparing to teach a client that metabolic syndrome can increase the risk for myocardial infarction (MI). Which signs of metabolic syndrome should the nurse include in the discussion? (Select all that apply.) Truncal obesity Hypercholesterolemia Elevated homocysteine levels Glucose intolerance Client taking losartan (Cozaar)

Truncal obesity Hypercholesterolemia Glucose intolerance Client taking losartan (Cozaar) A large waist size (excessive abdominal fat causing central obesity)—40 inches (102 cm) or greater for men, 35 inches (88 cm) or greater for women—is a sign of metabolic syndrome. Decreased high-density lipoprotein cholesterol (HDL-C) (usually with high low-density lipoprotein cholesterol)—HDL-C less than 40 mg/dL for men or less than 50 mg/dL for women—or taking an anticholesterol drug is a sign of metabolic syndrome. Increased fasting blood glucose (caused by diabetes, glucose intolerance, or insulin resistance) is included in the constellation of metabolic syndrome. Blood pressure greater than 130/85 mm Hg or taking antihypertensive medication indicates metabolic syndrome. Although elevated homocysteine levels may predispose to atherosclerosis, they are not part of metabolic syndrome.

A patient in the ED with chest pain has a possible MI. Which laboratory test is done to determine this diagnosis? a. Troponin T and I b. Serum potassium c. Homocysteine d. Highly sensitive C-reactive protein

a

A patient is being discharged with a prescription for warfarin (Coumadin). Which test does the nurse instruct the patient to routinely have done for follow-up monitoring? a. PT and INR b. PTT c. Complete blood count and platelet count d. Sodium and potassium levels

a

A patient's chart notes that the examiner has heard S1 and S2 on auscultation of the heart. What does this documentation refer to? a. First and second heart sounds b. Pericardial friction rub c. Murmur d. Gallop

a

Emergency personnel discovered a patient lying outside in the cool evening air for an unknown length of time. The patient is in a hypothermic state and the metabolic needs of the tissues are decreased. What other assessment finding does the nurse expect to see? a. Blood pressure and heart rate lower than normal b. Heart rate and respiratory rate higher than normal c. Normal vital signs due to compensatory mechanisms d. Gradually improved vital signs with enteral nutrition

a

In assessing a patient, the nurse finds that the PMI appears in more than one intercostal space, and has shifted laterally to the midclavicular line. How does the nurse interpret this data? a. Left ventricular hypertrophy b. Superior vena cava obstruction c. Pulmonary hypertension d. Constrictive pericarditis

a

The advanced-practice nurse is assessing the vascular status of a patient's lower extremities using the angle-brachial index. What is the correct technique for this assessment method? a. A blood pressure cuff is applied to the lower extremities and the systolic pressure is measured by Doppler ultrasound at both the dorsalis pedis and posterior tibial pulses b. The dorsalis pedis and posterior tibial pulses are manually palpated and compared bilaterally for strength and equality and compared to the standard of index c. A blood pressure cuff is applied to the lower extremities to observe for an exaggerated decrease in systolic pressure by more than 10 mm Hg during inspiration d. Measure blood pressure on the legs when the patient is supine; then have the patient stand fro several minutes and repeat blood pressure measurement in the arms

a

The nurse is caring for a patient at risk fro MI. For what primary reason does the nurse plan interventions to prevent anxiety or overexertion? a. An increase in heart rate increases myocardial oxygen demand b. A release of epinephrine and norepinephrine causes MI c. AN increase in activity or emotion affects preload and afterload d. Cardiac output is decreased by anxiety or physical stress

a

The nurse is performing a cardiac assessment on an older adult. What is a common assessment finding for this patient? a. S4 heart sound b. leg edema c. Pericardial friction rubs d. Change in point of maximum impulse location

a

The nurse is providing health teaching for a patient at risk for heart disease. Which factor is the most modifiable, controllable risk factor? a. Obesity b. Diabetes mellitus c. Ethnic background d. Family history of cardiovascular disease

a

The nurse is talking to a patient who has been trying to quit smoking. HWich statement by the patient indicates an understanding of cigarette usage as it relates to reducing cardiovascular risks? a. "I need to be completely cigarette-free for at least 3 years" b. "I don't smoke as much as I used to I'm down to one pack a day." c. "I started smoking a while ago, but I'll quit in a couple of years" d. "I only smoke to relax, when I drink, or when I go out with friends"

a

The nurse performing a physical assessment on a patient with a history of CVD observes that the patient has ascites, jaundice, and anasarca. How does the nurse interpret these findings? a. Late signs of severe right-sided heart failure b. Early signs of mild right-sided heart failure c. Late signs of mild left-sided heart failure d. Early signs of left- and right-sided heart failure

a

The nurse taking a medical history of a patient makes a special notation to follow up on valvular abnormalities of the heart. Which recurrent condition in the patient history causes the nurse to make this negotiation? a. Streptococcal infections of the throat b. Staphylococcal infections of the skin c. Vaginal yeast infections d. Fungal infections of the feet or inner thighs

a

The patient is scheduled for an exercise stress test. Which medications does the nurse expect the cardiologist will want held before the procedure? a. Atenolol and Cardizem b. Vitamins and potassium c. Colace and enteric-coated aspirin d. Acetaminophen and metered-dose bronchodilator

a

What is the most significant laboratory cardiac marker in a patient who has had an MI? a. Presence of troponin T and I b. Elevation of myoglobin levels c. Creatine kinase levels d. Elevation of the white blood cell count

a

When the nurse assesses a patient with CVD, there is difficulty auscultating the first heart sound (S1). What is the nurse's best action? a. Ask the patient to lean forward or roll to his or her left side b. Instruct the patient to take a deep breath and hold it c. Auscultate with the bell instead of the diaphragm d. Ask the UAP to complete a 12-lead ECG immediately

a

Which category of cardiovascular drug blocks the sympathetic stimulation to the heart and decreases the heart rate? a. Beta blockers b. Catecholamines c. Steroids d. Benzodiazepines

a

Which statement about the peripheral vascular system is true? a. Veins are equipped with valves that direct blood flow to the heart and prevent backflow b. The velocity of blood flow depends on the diameter of the vessel lumen c. Blood flow decreases and blood tends to clot as the viscosity decreases d. The parasympathetic nervous system has the largest effect on blood flow to organs

a

Which test is performed to determine valve disease of the mitral valve, left atrium, or aortic arch? a. Transesophageal echocardiogram b. ECG c. Myocardial nuclear perfusion imaging (MNPI) d. Phonocardiography

a

A client with unstable angina has received education about acute coronary syndrome. Which statement indicates that the client has understood the teaching? a. "This is a big warning; I must modify my lifestyle or risk having a heart attack in the next year." b. "Angina is just a temporary interruption of blood flow to my heart." c. "I need to tell my wife I've had a heart attack." d. "Because this was temporary, I will not need to take any medications for my heart."

a. "This is a big warning; I must modify my lifestyle or risk having a heart attack in the next year." Among people who have unstable angina, 10% to 30% have a myocardial infarction within 1 year. Although anginal pain is temporary, it reflects underlying coronary artery disease (CAD), which requires attention, including lifestyle modifications. Unstable angina reflects tissue ischemia, but infarction represents tissue necrosis. Clients with underlying CAD may need medications such as aspirin, lipid-lowering agents, anti-anginals, or antihypertensives.

When planning care for a client in the emergency department, which interventions are needed in the acute phase of myocardial infarction? (Select all that apply.) a. Morphine sulfate b. Oxygen c. Nitroglycerin d. Naloxone e. Acetaminophen f. Verapamil (Calan, Isoptin)

a. Morphine sulfate b. Oxygen c. Nitroglycerin Morphine is needed to reduce oxygen demand, preload, pain, and anxiety, and nitroglycerin is used to reduce preload and chest pain. Administering oxygen will increase available oxygen for the ischemic myocardium. Naloxone is a narcotic antagonist that is used for overdosage of opiates, not for MI. Acetaminophen may be used for headache related to nitroglycerin. Because of negative inotropic action, calcium channel blockers such as verapamil are used for angina, not for MI.

After receiving change-of-shift report in the coronary care unit, which client does the nurse assess first? a. The client with acute coronary syndrome who has a 3-pound weight gain and dyspnea b. The client with percutaneous coronary angioplasty who has a dose of heparin scheduled c. The client who had bradycardia after a myocardial infarction and now has a paced heart rate of 64 beats/min d. A client who has first-degree heart block, rate 68 beats/min, after having an inferior myocardial infarction

a. The client with acute coronary syndrome who has a 3-pound weight gain and dyspnea Dyspnea and weight gain are symptoms of left ventricular failure and pulmonary edema; this client needs prompt intervention. A scheduled heparin dose does not take priority over dyspnea; it can be administered after the client with dyspnea is taken care of. The client with a pacemaker and a normal heart rate is not in danger. First-degree heart block is rarely symptomatic, and the client has a normal heart rate; the client with dyspnea should be seen first.

The nurse is preparing to teach a client that metabolic syndrome can increase the risk for myocardial infarction (MI). Which signs of metabolic syndrome should the nurse include in the discussion? (Select all that apply.) a. Truncal obesity b. Hypercholesterolemia c. Elevated homocysteine levels d. Glucose intolerance e. Client taking losartan (Cozaar)

a. Truncal obesity b. Hypercholesterolemia d. Glucose intolerance e. Client taking losartan (Cozaar) A large waist size (excessive abdominal fat causing central obesity)—40 inches (102 cm) or greater for men, 35 inches (88 cm) or greater for women—is a sign of metabolic syndrome. Decreased high-density lipoprotein cholesterol (HDL-C) (usually with high low-density lipoprotein cholesterol)—HDL-C less than 40 mg/dL for men or less than 50 mg/dL for women—or taking an anticholesterol drug is a sign of metabolic syndrome. Increased fasting blood glucose (caused by diabetes, glucose intolerance, or insulin resistance) is included in the constellation of metabolic syndrome. Blood pressure greater than 130/85 mm Hg or taking antihypertensive medication indicates metabolic syndrome. Although elevated homocysteine levels may predispose to atherosclerosis, they are not part of metabolic syndrome.

Microalbuminuria has been shown to be a clear marker of widespread endothelial dysfunction in CVD. Which conditions should prompt patients to be tested annually for microalbuminuria? (SATA) a. Hypertension b. Metabolic syndrome c. Smoker d. Diabetes mellitus e. Use of anticoagulant therapy

abd

The nurse is providing discharge instructions for a patient who had a cardiac catheterization. Which instructions must the nurse include? (SATA) a. Notify the health care provider for increased swelling, redness, warmth, or pain b. Leave the dressing in place for the first day c. Limit activity for at least 2 to 3 weeks d. Avoid lifting and exercise for a few days e. Report any bruise or hematoma to the health care provider

abd

Which interventions and actions does the nurse perform to detect and prevent kidney toxicity when caring for a patient after cardiac catheterization? (SATA) a. Provide IV and oral fluids for 12 to 24 hours b. Monitor intake and output c. Check the catheterization site every hour d. Administer acetylcysteine if ordered e. Keep the catheterized extremity straight for 6 hours

abd

Which laboratory tests are used to predict a patient's risk for CAD? (SATA) a. Cholesterol level b. Triglyceride level c. Prothrombin time d. Low-density lipoprotein level e. Albumin level

abd

The nurse is giving a community presentation about heart disease in women. What information does the nurse include in the presentation? (SATA) a. Dyspnea on exertion may be the first and only symptoms of heart failure b. Symptoms are subtle or atypical c. Pain is often relieved by rest d. Having waist and abdominal obesity is a higher risk factor than having fat in buttocks and thighs e. Pain always responds to nitroglycerin f. Common symptoms include back pain, indigestion, nausea, vomiting, and anorexia

abdf

Which blood pressure reading require further assessment? (SATA) a. 90 mm Hg systolic b. 139 mm Hg Hg systolic c. 115 mm Hg systolic d 66 mm Hg diastolic e. 100 mm Hg diastolic

abe

The nurse is instructing a patient with CHF on what signs to look for when experiencing an exacerbation of CHF> Which are appropriate teaching points for this patient? (SATA) a. "It is possible to gain up to 10 or 15 lbs before edema develops" b. "Notify the provider of a weight loss of 3 to 5 lbs within 2 weeks" c. "Notify the provider of a weight gain of 2 lbs within 1 to 2 days" d. "Notify the provider if you notice that your shoes or rings feel tight" e. "Notify the provider if your skin becomes dry and scaly."

acd

The patient has a diagnosis of angina. Which assessment findings would the nurse expect to find? (SATA) a. Sudden onset of pain b. Intermittent pain relieved with sitting upright c. Substernal pain which may spread across chest, back, arms d. Pain usually lasts less than 15 minutes e. Sharp, stabbing pain which is moderate to severe

acd

Which are risk factors for CVD in women? (SATA) a. Waist and abdominal obesity b. Excess fat in the buttocks, hips, and thighs c. Postmenopausal d. Diabetes mellitus e. Asian ethnicity

acd

Which statements about blood pressure are accurate? (SATA) a. Pulse pressure is the difference between the systolic and diastolic pressures b. The right ventricle of the heart generates the greatest amount of blood pressure c. Diastolic blood pressure is primarily determined by the amount fo peripheral vasoconstriction d. To maintain adequate blood flow through the coronary arteries, mean arterial pressure (MAP) must be at least 60 mm Hg. e. Diastolic blood pressure is the highest pressure during contraction of the ventricles

acd

A patient comes to the ED reporting chest pain. In evaluating the patient's pain, which questions does the nurse ask the patient? (SATA) a. "How long does the pain last and how often does it occur?" b. "How do you feel about the pain?" c. "Is the pain different from any other episode of pain you've had?" d. "What activities were you doing when the pain first occurred?" e. "Where is the chest pain? What does it feel like?" f. "Have you had other signs and symptoms that occur at the same time?"

acdef

The nurse interprets a patient's serum lipid test. Which results suggest an increased risk for CVD? (SATA) a. LDL 160 mg/dL b HDL 60 mg/dL c. Total cholesterol 180 mg/dL d Triglycerides 175 mg/dL e Lp (a) 45 mg/dL

ade

The nurse is assessing a patient's nicotine dependency. Which questions does the nurse ask for an accurate assessment? (SATA) a. "How soon after you wake up in the morning do you smoke?" b. "What kind of cigarettes do you smoke?" c. "Do you inhale deeply when you smoke?" d. "Do you find it difficult not to smoke in places where smoking is prohibited?" e. "Do you smoke when you are ill?" f. "What happened the last time you tried to quit smoking"

adef

A patient entering the cardiac rehabilitation unit seems optimistic and at time unexpectedly cheerful and upbeat. Which statement by the patient causes the nurse to suspect a maladaptive use of denial in the patient? a. "I am sick and tired of talking about these dietary restriction. Could we talk about it tomorrow?" b. "Oh, I don't really need that medication information. I'm sure that I'll soon be able to get by without it" c. "This whole episode of heart problem has been an eye-opener for me, but I really can't wait to get out of her" d. "That doctor is really driving me crazy with all his instructions. Could you put all that information away in my suitcase?"

b

In a hypovolemic patient, stretch receptors in the blood vessels sense a reduced volume or pressure and send fewer impulses to the central nervous system. As a result, which signs/symptoms does the nurse expect to observe in the patient? a. Reddish mottling to skin and a blood pressure elevation b. Cool, pale skin and tachycardia c. Warm, flushed skin with low blood pressure d. Pale pink skin with bradycardia

b

In assessing a patient who has come to the clinic for a physical exam, the nurse sees that the patient has pallor. What is this finding most indicative of? a. Anemia b. Thrombophlebitis c. Heart disease d. Stroke

b

The health care provider orders orthostatic vital signs on a patient who experienced dizziness and feeling lightheaded. What is the nurse's first action a. Patient changes position to sitting or standing b. Measure the blood pressure when the patient is supine c. Place the patient in supine position for at least 3 minutes d. Wait for at least 1 minute before auscultating blood pressure and counting the radial pulse

b

The nurse is performing an assessment on a patient brought in by emergency personnel. The nurse immediately observes that the patient has spontaneous respirations and the skin is cool, pale, and moist. What is the priority patient problem? a. Abnormal body temperature b. Impaired oxygenation c. Altered skin integrity d. Potential for peripheral neurovascular dysfunction

b

The nurse working in a women's health clinic is reviewing the risk factors for several patients for stroke and MI. Which patient has the highest risk for MI? a. 49-year-old on estrogen replacement therapy b. 40-year-old taking oral contraceptives who smokes c. 23-year-old with diabetes that is currently not well-controlled d. 60-year-old with well-controlled hypertension

b

The nurse working in the public health department is reviewing data for populations at risk fro CVD. Which group has the greatest need for intervention to reduce CVD risk? a. Government employees who make approximately $40,000 a year b. Part-time fast-food workers who make approximately $9,000 a year c. Hospital-employed nurses who make approximately $52,000 a year d. Chain retail employees who make approximately $18,000 a year

b

The patient has smoked half a pack of cigarettes per day for 2 years. How many pack-years has this patient smoked? a. 1/2 pack year b. 1 back year c. 1 1/5 pack years d. 2 pack years

b

The patient with a history of allergy to iodine-based contrast dyes is scheduled for a cardiac catheterization. What action does the nurse expect with regard to the scheduled test? a. Delay the test for a week or more b. Administer an antihistamine and.or steroid before the test c. The test will be performed without administration of contrast dye d. The patient will receive anticoagulation therapy before the test

b

What is the correct technique for assessing a patient with arterial insufficiency in the right lower leg? a. Use the Doppler to find the dorsalis pedis and posterior tibial pulses on the right leg b. Palpate the peripheral arteries in a head-to-toe approach with a side-to-side comparison c. Check all the pulse points in the right leg independent and supine positions d. Palpate the major arteries, such as the radial and femoral, and observe for pallor

b

What is the significance of a sodium level of 130 mEq/L for a patient with heart failure? a. Increased risk for ventricular dysrhythmias b. Dilutional hyponatremia and fluid retention c. Potential for electrical instability of the heart d. Slowed conduction of impulse through the heart

b

What term describes the difference between systolic and diastolic values, which is an indirect measure of cardiac output? a. Paradoxical blood pressure b. Pulse pressure c. Ankle-brachial index d. Normal blood pressure

b

Which assessment finding in a patient who has had a cardiac catheterization does the nurse report immediately to the provider? a. Pain at the catheter insertion site b. Catheterized extremity dusky with decreased peripheral pulses c. Small hematoma at the catheter insertion site d. Pulse pressure of 40 mm Hg with a slow, bounding pulse

b

Which description best defines the cardiovascular concept afterload? a. Degree of myocardial fiber stretch at end of diastole and just before heart contracts b. Amount of pressure or resistance that the ventricles must overcome to eject blood through the semilunar valves and into the peripheral blood vessels c. Pressure that ventricle must overcome to open aortic valve d. Force of contraction independent of preload

b

The nurse is caring for a group of clients who have sustained myocardial infarction (MI). The nurse observes the client with which type of MI most carefully for the development of left ventricular heart failure? a. Inferior wall b. Anterior wall c. Lateral wall d. Posterior wall

b. Anterior wall Due to the large size of the anterior wall, the amount of tissue infarction may be large enough to decrease the force of contraction, leading to heart failure. The client with an inferior wall MI is more likely to develop right ventricular heart failure. Clients with obstruction of the circumflex artery may experience a lateral wall MI and sinus dysrhythmias or a posterior wall MI and sinus dysrhythmias.

A client has just returned from coronary artery bypass graft surgery. For which finding does the nurse contact the surgeon? a. Temperature 98.2° F b. Chest tube drainage 175 mL last hour c. Serum potassium 3.9 mEq/L d. Incisional pain 6 on a scale of 0 to 10

b. Chest tube drainage 175 mL last hour Some bleeding is expected after surgery; however, the nurse should report chest drainage over 150 mL/hr to the surgeon. Although hypothermia is a common problem after surgery, a temperature of 98.2° F is a normal finding. Serum potassium of 3.9 mEq/L is a normal finding. Incisional pain of 6 on a scale of 0 to 10 is expected immediately after major surgery; the nurse should administer prescribed analgesics.

The nurse in the coronary care unit is caring for a group of clients who have had myocardial infarction. Which client does the nurse see first? a. Client with dyspnea on exertion when ambulating to the bathroom b. Client with third-degree heart block on the monitor c. Client with normal sinus rhythm and PR interval of 0.28 second d. Client who refuses to take heparin or nitroglycerin

b. Client with third-degree heart block on the monitor Third-degree heart block is a serious complication that indicates that a large portion of the left ventricle and conduction system are involved, so the client with the third-degree heart block should be seen first. Third-degree heart block usually requires pacemaker insertion. A normal rhythm with prolonged PR interval indicates first-degree heart block, which usually does not require treatment. The client with dyspnea on exertion when ambulating to the bathroom is not at immediate risk. The client's uncooperative behavior when refusing to take heparin or nitroglycerin may indicate fear or denial; he should be seen after emergency situations have been handled.

The nurse is concerned that a client who had myocardial infarction (MI) has developed cardiogenic shock. Which findings indicate shock? (Select all that apply.) a. Bradycardia b. Cool, diaphoretic skin c. Crackles in the lung fields d. Respiratory rate of 12 breaths/min e. Anxiety and restlessness f. Temperature of 100.4° F

b. Cool, diaphoretic skin c. Crackles in the lung fields e. Anxiety and restlessness The client with shock has cool, moist skin. Because of extensive tissue necrosis, the left ventricle cannot forward blood adequately, resulting in pulmonary congestion and crackles. Because of poor tissue perfusion, a change in mental status, anxiety, and restlessness are expected. All types of shock (except neurogenic) present with tachycardia, not bradycardia. Due to pulmonary congestion, a client with cardiogenic shock typically has tachypnea. Cardiogenic shock does not present with low-grade fever; this would be more likely to occur in pericarditis.

The client in the cardiac care unit has had a large myocardial infarction. How does the nurse recognize onset of left ventricular failure? a. Urine output of 1500 mL on the preceding day b. Crackles in the lung fields c. Pedal edema d. Expectoration of yellow sputum

b. Crackles in the lung fields Manifestations of left ventricular failure and pulmonary edema are noted by listening for crackles and identifying their locations in the lung fields. A urine output of 1500 mL is normal. Edema is a sign of right ventricular heart failure. Yellow sputum indicates the presence of white blood cells and possible infection.

Which atypical symptoms may be present in a female client experiencing myocardial infarction (MI)? (Select all that apply.) a. Sharp, inspiratory chest pain b. Dyspnea c. Dizziness d. Extreme fatigue e. Anorexia

b. Dyspnea c. Dizziness d. Extreme fatigue Many women who experience an MI present with dyspnea, light-headedness, and fatigue. Sharp, pleuritic pain is more consistent with pericarditis or pulmonary embolism. Anorexia is neither a typical nor an atypical sign of MI.

After thrombolytic therapy, the nurse working in the cardiac catheterization laboratory would be alarmed to notice which sign? a. A 1-inch backup of blood in the IV tubing b. Facial drooping c. Partial thromboplastin time (PTT) 68 seconds d. Report of chest pressure during dye injection

b. Facial drooping During and after thrombolytic administration, the nurse observes for any indications of bleeding, including changes in neurologic status, which may indicate intracranial bleeding. A 1-inch backup of blood in the IV tubing may be related to IV positioning. If heparin is used, PTT reflects a therapeutic value. Reports of chest pressure during dye injection or stent deployment are considered an expected result of the procedure.

Which characteristics place women at high risk for myocardial infarction (MI)? (Select all that apply.) a. Premenopausal b. Increasing age c. Family history d. Abdominal obesity e. Breast cancer

b. Increasing age c. Family history d. Abdominal obesity Increasing age is a risk factor, especially after 70 years. Family history is a significant risk factor in both men and women. A large waist size and/or abdominal obesity are risk factors for both metabolic syndrome and MI. Premenopausal women are not at higher risk for MI, and breast cancer is not a risk factor for MI.

A client comes to the emergency department with chest discomfort. Which action does the nurse perform first? a. Administers oxygen therapy b. Obtains the client's description of the chest discomfort c. Provides pain relief medication d. Remains calm and stays with the client

b. Obtains the client's description of the chest discomfort A description of the chest discomfort must be obtained first, before further action can be taken. Neither oxygen therapy nor pain medication is the first priority in this situation; an assessment is needed first. Remaining calm and staying with the client are important, but are not matters of highest priority.

Prompt pain management with myocardial infarction is essential for which reason? a. The discomfort will increase client anxiety and reduce coping. b. Pain relief improves oxygen supply and decreases oxygen demand. c. Relief of pain indicates that the MI is resolving. d. Pain medication should not be used until a definitive diagnosis has been established.

b. Pain relief improves oxygen supply and decreases oxygen demand. The focus of pain relief is on reducing myocardial oxygen demand. Chest discomfort will increase anxiety, but it may not affect coping. Relief of pain is secondary to the use of opiates or indicates that the tissue infarction is complete. Although it used to be true that pain medication was not to be used for undiagnosed abdominal pain, this does not relate to MI.

Syncope in the aging person can likely occur with which actions by the patient? (SATA) a. Laughing b. Turning the head c. Performing a Valsalva maneuver d. Walking briskly for 20 to 30 minutes e. shrugging the shoulders

bce

The nurse is caring for a patient at risk for heart problems. What are normal findings for the cardiovascular assessment of this patient? (SATA) a. Presence of a thrill b. Splitting of S2; decreases with expiration c. Jugular venous distention to level of the mandible d. Point of maximal impulse (PMI) in fifth intercostal space at midclavicular line e. Paradoxical chest movement with inspiration and expiration

bd

What are the purposes of angiography? (SATA) a. Determine an abnormal structure of the heart b. Identify an arterial obstruction c. Assess the cardiovascular response to increased workload d. Identify an arterial narrowing e. Identify an aneurysm

bde

What different pathophysiologic conditions can the health heart adapt to? (SATA) a. Menses b. Stress c. Gastroesophageal reflux disease d. Infection e. Hemorrhage

bde

The nurse is assessing a 62-year-old native Hawaiian woman. She is postmenopausal, diabetic fro 10 years, smokes 1 pack a day of cigarettes for 20 years, walks twice a week for 30 minutes, is an administrator, and describes her lifestyle as sedentary. For her weight and height she has a body mass index of 32. Which risk factors for this patient are controllable for CVD? (SATA) a. Ethnic background b. Smoking c. Age d. Obesity e. Postmenopausal f. Sedentary lifestyle

bdf

What measures are taken to prepare a patient for a pharmacologic stress echocardiogram? (SATA) a. Patient can eat his/her diet as ordered b. IV access needs to be present c. Oxygen at 2 L per nasal cannula is placed on patient 3 hours prior to test d. An oral laxative is given the day before the test e. Patient is to be NPO for 3 to 6 hours before the test

be

A patient comes to the clinic stating "my right foot turns a darkish red color when I sit too long, and when I put my foot up, it turns pale." Which conditions does the nurse suspect? a. Central cyanosis b. Peripheral cyanosis c. Arterial insufficiency d. Venous insufficiency

c

A patient is scheduled to have an exercise electrocardiography test. What instruction does the nurse provide to the patient before the procedure takes place? a. "Have nothing to eat or drink after midnight." b. "Avoid smoking or drinking alcohol for at least 2 weeks before the test" c "Wear comfortable, loose clothing and rubber-soled, supportive shoes" d "Someone must drive you home because of possible sedative effects of the medications"

c

A patient is undergoing diagnostic testing for reports of chest pain. Which test is done to determine the location and extent of CAD? a. ECG b. Echocardiogram c. Cardiac catheterization d. Chest x-ray

c

A young patient reports having frequent episodes of palpitations but denies having chest pain. Which follow-up question does the nurse ask to assess the patient symptom of palpitations? a. "Have you noticed a worsening of shortness of breath when you are lying flat?" b "Do your shoes feel unusually tight, or are your rings tighter than usual?" c. "Do you feel dizzy or have you lost consciousness with the palpitations?" d. "Does anyone in your family have a history of palpitations?"

c

Based on the physiologic force that propels blood forward in the veins, the which patient has the greatest risk for venous stasis? a. Older adult patient with hypertension who rides a bicycle daily b. Middle-aged construction worker taking Coumadin c. Bedridden patient in the end stage of Alzheimer's disease d. Teenage patient with a broken leg who sits and plays videogames

c

The nurse is caring for a patient with the priority problem of decreased cardiac output. Which situation may result in decreased myocardial contractility that will further lower cardiac output? a. Administration of a positive inotropic medication b. Hyperventilation to correct respiratory acidosis c. Frequent endotracheal suctioning that result in hypoxemia d. Administration of IV fluids to correct underlying hypovolemia

c

The nurse is taking report on a client who will be transferred from the cardiac intensive care unit to the general medical-surgical unit. The reporting nurse states that S4 is heard on auscultation of the heart. This indicates that the patient has which condition? a. Heart murmur b. Pericardial friction rub c. Ventricular hypertrophy d. Normal heart sounds

c

The nurse practitioner reads in a patient's chart that a carotid bruit was heard during the last two annual checkups. Today on auscultation, the bruit is not present. How does the nurse practitioner evaluate this data? a. The problem has resolved spontaneously b. There may have been an anomaly in previous findings c. The occlusion of the vessel may have progressed past 90% d. The antiplatelet therapy is working

c

Which category of cardiovascular drugs increases heart rate and contractility? a. Diuretics b. Beta blockers c. Catecholamines d. Benzodiazepines

c

a 65-year-old patient comes to the clinic reporting fatigue. The patient would like to start an exercise program, but thinks "anemia might be causing the fatigue." What is the nurse's first action? a. Advise the patient to start out slowly and gradually build strength and endurance b. Obtain an order for a complete blood count and nutritional profile c. Assess the onset, duration, and circumstances associated with the fatigue d. Perform a physical assessment to include testing of muscle strength and tone.

c

A client undergoing coronary artery bypass grafting asks why the surgeon has chosen to use the internal mammary artery for the surgery. Which response by the nurse is correct? a. "This way you will not need to have a leg incision." b. "The surgeon prefers this approach because it is easier." c. "These arteries remain open longer." d. "The surgeon has chosen this approach because of your age."

c. "These arteries remain open longer." Mammary arteries remain patent much longer than other grafts. Although no leg incision will be made with this approach, veins from the legs do not remain patent as long as the mammary artery graft does. Long-term patency, not ease of the procedure, is the primary concern. Age is not a determining factor in selection of these grafts.

An LPN/LVN is scheduled to work on the inpatient "stepdown" cardiac unit. Which client does the charge nurse assign to the LPN/LVN? a. A 60-year-old who was admitted today for pacemaker insertion because of third-degree heart block and who is now reporting chest pain b. A 62-year-old who underwent open heart surgery 4 days ago for mitral valve replacement and who has a temperature of 38.2° C c. A 66-year-old who has a prescription for a nitroglycerin (Nitro-Dur) patch and is scheduled for discharge to a group home later today d. A 69-year-old who had a stent placed 2 hours ago in the left anterior descending artery and who has bursts of ventricular tachycardia

c. A 66-year-old who has a prescription for a nitroglycerin (Nitro-Dur) patch and is scheduled for discharge to a group home later today The LPN/LVN scope of practice includes administration of medications to stable clients. Third-degree heart block is characterized by a very low heart rate and usually by required pacemaker insertion; the skills of the RN are needed to care for this client. Fever after surgery requires collaboration with the health care provider, which is more consistent with the role of the RN. The client with a recent stent placement and having bursts of ventricular tachycardia is unstable and is showing ventricular irritability; he will need medications and monitoring beyond the scope of practice of the LPN/LVN.

An older adult client, 4 hours after coronary artery bypass graft (CABG), has a blood pressure of 80/50 mm Hg. What action does the nurse take? a. No action is required; low blood pressure is normal for older adults. b. No action is required for postsurgical CABG clients. c. Assess pulmonary artery wedge pressure (PAWP). d. Give ordered loop diuretics.

c. Assess pulmonary artery wedge pressure (PAWP). Decreased preload as exhibited by decreased PAWP could indicate hypovolemia secondary to hemorrhage or vasodilation; hypotension could cause the graft to collapse. Low blood pressure is not normal in older adults or postoperative clients. The cause of hypotension must be found and treated; further action is needed to determine additional interventions. Hypotension could be caused by hypovolemia; giving loop diuretics increases hypovolemia.

The nurse is teaching a group of teens about prevention of heart disease. Which point should the nurse emphasize? a. Reduce abdominal fat. b. Avoid stress. c. Do not smoke or chew tobacco. d. Avoid alcoholic beverages.

c. Do not smoke or chew tobacco. Tobacco exposure, including secondhand smoke, reduces coronary blood flow; causes vasoconstriction, endothelial dysfunction, and thickening of the vessel walls; increases carbon monoxide; and decreases oxygen. Because it is highly addicting, beginning smoking in the teen years may lead to decades of exposure. Teens are not likely to experience metabolic syndrome from obesity, but are very likely to use tobacco. Avoiding stress is a less modifiable risk factor, which is less likely to cause heart disease in teens. The risk of smoking outweighs the risk of alcohol use. Tobacco exposure, including secondhand smoke, reduces coronary blood flow; causes vasoconstriction, endothelial dysfunction, and thickening of the vessel walls; increases carbon monoxide; and decreases oxygen. Because it is highly addicting, beginning smoking in the teen years may lead to decades of exposure. Teens are not likely to experience metabolic syndrome from obesity, but are very likely to use tobacco. Avoiding stress is a less modifiable risk factor, which is less likely to cause heart disease in teens. The risk of smoking outweighs the risk of alcohol use.

The visiting nurse is seeing a client postoperative for coronary artery bypass graft. Which nursing action should be performed first? a. Assess coping skills. b. Assess for postoperative pain at the client's incision site. c. Monitor for dysrhythmias. d. Monitor mental status.

c. Monitor for dysrhythmias. Dysrhythmias are the leading cause of prehospital death; the nurse should monitor the client's heart rhythm. Assessing mental status, coping skills, or postoperative pain is not the priority for this client.

The nurse is caring for a client in phase 1 cardiac rehabilitation. Which activity does the nurse suggest? a. The need to increase activities slowly at home b. Planning and participating in a walking program c. Placing a chair in the shower for independent hygiene d. Consultation with social worker for disability planning

c. Placing a chair in the shower for independent hygiene Phase 1 begins with the acute illness and ends with discharge from the hospital; it focuses on promoting rest and allowing clients to improve their activities of daily living based on their abilities. Phase 2 begins after discharge and continues through convalescence at home, including consultation with a social worker for long-term planning; it consists of achieving and maintaining a vital and productive life while remaining within the limits of the heart's ability to respond to increases in activity and stress. Phase 3 refers to long-term conditioning, such as a walking program.

What is included in postprocedural care of a patient after a cardiac catheterization? (SATA) a. Patient remains on bedrest for 12 to 24 hours b. Patient is placed in a high-Fowlers position c. Dressing is assessed for bloody drainage or hematoma d. Peripheral pulses in the affected extremity, as well as skin temperature and color, are monitored with every vital sign check e. Adequate oral and IV fluids are provided for hydration f. Vital signs are monitored every hour for 24 hours

cde

A nurse is monitoring the patient blood pressure and ECG during a stress test. Which parameter indicates the patient should stop exercising? a. Increase in heart rate b. Increase in blood pressure c. ECG showing the PQRS complex d. ECG showing St-segment depression

d

A patient reports severe cramping in the legs while attempting to walk for exercise. The provider diagnoses the patient with intermittent claudication. What does the nurse advise the patient to do? a. Elevate the legs on a pillow b. Buy and wear supportive shoes c. Massage the legs before walking d. Rest the legs in a dependent position

d

The nurse is assessing a patient with suspected CVD. When assessing the precordium, which assessment technique does the nurse begin? a. Percussion b. Palpation c. Auscultation d. Inspection

d

The nurse is performing a dietary assessment on a 45-year-old business executive at risk for CVD. Which assessment method used by the nurse is the most reliable and accurate? a. Ask the patient to identify foods he or she eats that contain sodium, sugar, cholesterol, fiber, and fat b. Ask the patient's spouse, who does the cooking and shopping, to identify the types of foods that are consumed c. Ask the patient how cultural beliefs and economic status influence the choice of food items d. Ask the patient to recall the intake of food, fluids, and alcohol during a typical 24-hour period

d

Which exercise regimen for an older adult meets the recommended guidelines of physical fitness to promote heart health? a. 6-hour bike ride every Saturday b. Golfing for 4 hours two times a week c. Running for 15 minutes three times a week d. Brisk walk 30 minutes every day

d

Which medications will the nurse be sure to hold until after a patient's cardiac catheterization? a. Daily vitamin and enteric coated aspirin b. Atenolol and IV antibiotic c. Potassium and folic acid d. Digoxin and furosemide

d

Which patient has an abnormal heart sound? a. S1 in a 45-year-old pateint b. S2 in a 30-year-old patient c. S3 in a 15-year-old patient d. S3 in a 54-year-old patient

d

While listening to a patient's heart sounds, the nurse detects a murmur. What does the nurse understand about the cause of murmurs? a. A murmur is caused by the closing of the aortic and pulmonic valves b. A murmur is caused when blood flows from the atrium to a noncompliant ventricle c. A murmur is caused by anemia, hypertension, or ventricular hypertrophy d. A murmur is caused when there is turbulent blood flow through normal or abnormal valves

d

Which statement by a client scheduled for a percutaneous transluminal coronary angioplasty (PTCA) indicates a need for further preoperative teaching? a. "I will be awake during this procedure." b. "I will have a balloon in my artery to widen it." c. "I must lie still after the procedure." d. "My angina will be gone for good."

d. "My angina will be gone for good." Reocclusion is possible after PTCA. The client is typically awake, but drowsy, during this procedure. PTCA uses a balloon to widen the artery, and the client will have to lie still after the procedure because of the large-bore venous access. Time is necessary to allow the hole to heal and prevent hemorrhage.

During discharge planning after admission for a myocardial infarction, the client says, "I won't be able to increase my activity level. I live in an apartment, and there is no place to walk." What is the nurse's best response? a. "You are right. Work on your diet then." b. "You must find someplace to walk." c. "Walk around the edge of your apartment complex." d. "Where might you be able to walk?"

d. "Where might you be able to walk?" Asking the client where he or she might be able to walk calls for cooperation and participation from the client; increased activity is imperative for this client. Telling the client to work on diet is an inappropriate response. Telling the client to find someplace to walk is too demanding to be therapeutic. Telling the client to walk around the apartment complex is domineering and will not likely achieve cooperation from the client.

To validate that a client has had a myocardial infarction (MI), the nurse assesses for positive findings on which tests? a. Creatine kinase-MB fraction (CK-MB) and alkaline phosphatase b. Homocysteine and C-reactive protein c. Total cholesterol, low-density lipoprotein cholesterol, and high-density lipoprotein cholesterol d. CK-MB and troponin

d. CK-MB and troponin CK-MB and troponin are the cardiac markers used to determine whether MI has occurred. Alkaline phosphatase is often elevated in liver disease. Homocysteine and C-reactive protein are markers of inflammation, which may represent risk for MI, but they are not diagnostic for MI. Elevated cholesterol levels are risks for MI, but they do not validate that an MI has occurred.

The nurse is caring for a client 36 hours after coronary artery bypass grafting, with a priority problem of intolerance for activity related to imbalance of myocardial oxygen supply and demand. Which finding causes the nurse to terminate an activity and return the client to bed? a. Pulse 60 beats/min and regular b. Urinary frequency c. Incisional discomfort d. Respiratory rate 28 breaths/min

d. Respiratory rate 28 breaths/min Tachypnea and tachycardia reflect activity intolerance; activity should be terminated. Pulse 60 beats/min and regular is a normal finding. Urinary frequency may indicate infection or diuretic use, but not activity intolerance. Pain with activity after surgery is anticipated; pain medication should be available.

The nurse is assessing a client with chest pain to evaluate whether the client is suffering from angina or myocardial infarction (MI). Which symptom is indicative of an MI? a. Chest pain brought on by exertion or stress b. Substernal chest discomfort occurring at rest c. Substernal chest discomfort relieved by nitroglycerin or rest d. Substernal chest pressure relieved only by opioids

d. Substernal chest pressure relieved only by opioids Substernal chest pressure relieved only by opioids is typically indicative of MI. Substernal chest discomfort that occurs at rest is not necessarily indicative of MI; it could be a sign of unstable angina. Both chest pain brought on by exertion or stress and substernal chest discomfort relieved by nitroglycerin or rest are indicative of angina.

do online practice questions

do online practice questions

do study guide practice questions

do study guide practice questions

Which statement by a client scheduled for a percutaneous transluminal coronary angioplasty (PTCA) indicates a need for further preoperative teaching? "I will be awake during this procedure." "I will have a balloon in my artery to widen it." "I must lie still after the procedure." "My angina will be gone for good."

"My angina will be gone for good." Reocclusion is possible after PTCA. The client is typically awake, but drowsy, during this procedure. PTCA uses a balloon to widen the artery, and the client will have to lie still after the procedure because of the large-bore venous access. Time is necessary to allow the hole to heal and prevent hemorrhage.

A client undergoing coronary artery bypass grafting asks why the surgeon has chosen to use the internal mammary artery for the surgery. Which response by the nurse is correct? "This way you will not need to have a leg incision." "The surgeon prefers this approach because it is easier." "These arteries remain open longer." "The surgeon has chosen this approach because of your age."

"These arteries remain open longer." Mammary arteries remain patent much longer than other grafts. Although no leg incision will be made with this approach, veins from the legs do not remain patent as long as the mammary artery graft does. Long-term patency, not ease of the procedure, is the primary concern. Age is not a determining factor in selection of these grafts.

A client with unstable angina has received education about acute coronary syndrome. Which statement indicates that the client has understood the teaching? "This is a big warning; I must modify my lifestyle or risk having a heart attack in the next year." "Angina is just a temporary interruption of blood flow to my heart." "I need to tell my wife I've had a heart attack." "Because this was temporary, I will not need to take any medications for my heart."

"This is a big warning; I must modify my lifestyle or risk having a heart attack in the next year." Among people who have unstable angina, 10% to 30% have a myocardial infarction within 1 year. Although anginal pain is temporary, it reflects underlying coronary artery disease (CAD), which requires attention, including lifestyle modifications. Unstable angina reflects tissue ischemia, but infarction represents tissue necrosis. Clients with underlying CAD may need medications such as aspirin, lipid-lowering agents, anti-anginals, or antihypertensives.

During discharge planning after admission for a myocardial infarction, the client says, "I won't be able to increase my activity level. I live in an apartment, and there is no place to walk." What is the nurse's best response? "You are right. Work on your diet then." "You must find someplace to walk." "Walk around the edge of your apartment complex." "Where might you be able to walk?"

"Where might you be able to walk?" Asking the client where he or she might be able to walk calls for cooperation and participation from the client; increased activity is imperative for this client. Telling the client to work on diet is an inappropriate response. Telling the client to find someplace to walk is too demanding to be therapeutic. Telling the client to walk around the apartment complex is domineering and will not likely achieve cooperation from the client.

30. Which client teaching should the nurse implement for the client diagnosed with coronary artery disease? Select all that apply. 1. Encourage a low-fat, low-cholesterol diet. 2. Instruct the client to walk 30 minutes a day. 3. Decrease the salt intake to two (2) g a day. 4. Refer to a counselor for stress reduction techniques. 5. Teach the client to increase fiber in the diet.

1. A low-fat, low-cholesterol diet will help decrease the buildup of atherosclerosis in the arteries. 2. Walking will help increase collateral circulation. 4. Stress reduction is encouraged for clients with CAD because this helps prevent excess stress on the heart muscle. 5. Increasing fiber in the diet will help remove cholesterol via the gastrointestinal system. TEST-TAKING HINT: This is an alternate-type question where the test taker must select all interventions that are applicable to the situation. Coronary artery disease is a common disease, and the nurse must be knowledgeable about ways to modify risk factors.

20. The intensive care department nurse is assessing the client who is 12 hours post-myocardial infarction. The nurse assesses an S3 heart sound. Which intervention should the nurse implement? 1. Notify the health-care provider immediately. 2. Elevate the head of the client's bed. 3. Document this as a normal and expected finding. 4. Administer morphine intravenously.

1. An S3 indicates left ventricular failure and should be reported to the health-care provider. It is a potential life-threatening complication of a myocardial infarction. TEST-TAKING HINT: There are some situations in which the nurse must notify the healthcare provider, and the test taker should not automatically eliminate this as a possible correct answer. The test taker must decide if any of the other three options will help correct a life-threatening complication. Normal assessment concepts should help identify the correct option. The normal heart sounds are S1 and S2 ("lubb-dupp"); S3 is abnormal.

34. The client with coronary artery disease is prescribed a Holter monitor. Which intervention should the nurse implement? 1. Instruct the client to keep a diary of activity, especially when having chest pain. 2. Discuss the need to remove the Holter monitor during a.m. care and showering. 3. Explain that all medications should be withheld while wearing a Holter monitor. 4. Teach the client the importance of decreasing activity while wearing the monitor.

1. The Holter monitor is a 24-hour electrocardiogram, and the client must keep an accurate record of activity so that the health-care provider can compare the ECG recordings with different levels of activity. TEST-TAKING HINT: In some instances, the test taker must be knowledgeable about diagnostic tests and there are no test-taking hints. The test taker might eliminate option "3" by realizing that, unless the client is NPO for a test or surgery, medications are usually taken.

23. The client diagnosed with a myocardial infarction asks the nurse, "Why do I have to rest and take it easy? My chest doesn't hurt anymore." Which statement would be the nurse's best response? 1. "Your heart is damaged and needs about four (4) to six (6) weeks to heal." 2. "There is necrotic myocardial tissue that puts you at risk for dysrhythmias." 3. "Your doctor has ordered bedrest. Therefore, you must stay in the bed." 4. "Just because your chest doesn't hurt anymore doesn't mean you are out of danger."

1. The heart tissue is dead, stress or activity may cause heart failure, and it does take about six (6) weeks for scar tissue to form. TEST-TAKING HINT: When attempting to answer a client's question, the nurse should provide factual information in simple, understandable terms. The test taker should select the answer option that provides this type of information.

26. The client with coronary artery disease asks the nurse, "Why do I get chest pain?" Which statement would be the most appropriate response by the nurse? 1. "Chest pain is caused by decreased oxygen to the heart muscle." 2. "There is ischemia to the myocardium as a result of hypoxemia." 3. "The heart muscle is unable to pump effectively to perfuse the body." 4. "Chest pain occurs when the lungs cannot adequately oxygenate the blood."

1. This is a correct statement presented in layman's terms. When the coronary arteries cannot supply adequate oxygen to the heart muscle, there is chest pain. TEST-TAKING HINT: The nurse must select the option that best explains the facts in terms a client who does not have medical training can understand.

28. The nurse is preparing to administer a beta blocker to the client diagnosed with coronary artery disease. Which assessment data would cause the nurse to question administering the medication? 1. The client has a BP of 110/70. 2. The client has an apical pulse of 56. 3. The client is complaining of a headache. 4. The client's potassium level is 4.5 mEq/L.

2. A beta blocker decreases sympathetic stimulation to the heart, thereby decreasing the heart rate. An apical rate less than 60 indicates a lower-than-normal heart rate and should make the nurse question administering this medication because it will further decrease the heart rate. TEST-TAKING HINT: If the test taker does not know when to question the use of a certain medication, the test taker should evaluate the options to determine if any options include abnormal data based on normal parameters. This would make the test taker select option "2" because the normal apical pulse in an adult is 60 to 100.

16. The nurse is caring for a client diagnosed with a myocardial infarction who is experiencing chest pain. Which interventions should the nurse implement? Select all that apply. 1. Administer morphine intramuscularly. 2. Administer an aspirin orally. 3. Apply oxygen via a nasal cannula. 4. Place the client in a supine position. 5. Administer nitroglycerin subcutaneously.

2. Aspirin is an antiplatelet medication and should be administered orally. 3. Oxygen will help decrease myocardial ischemia, thereby decreasing pain. TEST-TAKING HINT: This is an alternate-type question that requires the test taker to select all options that are applicable. The test taker must identify all correct answer options to receive credit for a correct answer; no partial credit is given. Remember to read the question carefully—it is not meant to be tricky.

14. Along with persistent, crushing chest pain, which signs/symptoms would make the nurse suspect that the client is experiencing a myocardial infarction? 1. Midepigastric pain and pyrosis. 2. Diaphoresis and cool, clammy skin. 3. Intermittent claudication and pallor. 4. Jugular vein distention and dependent edema.

2. Diaphoresis (sweating) is a systemic reaction to the MI. The body vasoconstricts to shunt blood from the periphery to the trunk of the body; this, in turn, leads to cold, clammy skin. TEST-TAKING HINT: The stem already addresses chest pain; therefore, the test taker could eliminate option "1" as a possible answer. Intermittent claudication, option "3," is the classic sign of arterial occlusive disease, and JVD is very specific to congestive heart failure. The nurse must be able to identify at least two or three signs/symptoms of disease processes.

29. Which intervention should the nurse implement when administering a loop diuretic to a client diagnosed with coronary artery disease? 1. Assess the client's radial pulse. 2. Assess the client's serum potassium level. 3. Assess the client's glucometer reading. 4. Assess the client's pulse oximeter reading.

2. Loop diuretics cause potassium to be lost in the urine output. Therefore, the nurse should assess the client's potassium level, and if the client is hypokalemic, the nurse should question administering this medication. TEST-TAKING HINT: Knowing that diuretics increase urine output would lead the test taker to eliminate glucose level and oxygenation (options "3" and "4"). In very few instances does the nurse assess the radial pulse; the apical pulse is assessed.

33. The nurse is discussing angina with a client who is diagnosed with coronary artery disease. Which action should the client take first when experiencing angina? 1. Put a nitroglycerin tablet under the tongue. 2. Stop the activity immediately and rest. 3. Document when and what activity caused angina. 4. Notify the health-care provider immediately.

2. Stopping the activity decreases the heart's need for oxygen and may help decrease the angina (chest pain). TEST-TAKING HINT: The question is asking which action the client should take first. This implies that more than one of the answer options could be appropriate for the chest pain, but that only one is done first. The test taker should select the answer that will help the client directly and quickly—and that is stopping the activity.

35. Which statement by the client diagnosed with coronary artery disease indicates that the client understands the discharge teaching concerning diet? 1. "I will not eat more than six (6) eggs a week." 2. "I should bake or grill any meats I eat." 3. "I will drink eight (8) ounces of whole milk a day." 4. "I should not eat any type of pork products."

2. The American Heart Association recommends a low-fat, low-cholesterol diet for a client with coronary artery disease. The client should avoid any fried foods, especially meats, and bake, broil, or grill any meat. TEST-TAKING HINT: The test taker must be knowledgeable of prescribed diets for specific disease processes. This is mainly memorizing facts. There is no test-taking hint to help eliminate any of the options.

18. The client is one (1) day postoperative coronary artery bypass surgery. The client complains of chest pain. Which intervention should the nurse implement first? 1. Medicate the client with intravenous morphine. 2. Assess the client's chest dressing and vital signs. 3. Encourage the client to turn from side to side. 4. Check the client's telemetry monitor.

2. The nurse must always assess the client to determine if the chest pain that is occurring is expected postoperatively or if it is a complication of the surgery. TEST-TAKING HINT: The stem asks the nurse to identify the first intervention that should be implemented. Therefore, the test taker should apply the nursing process and select an assessment intervention. Both options "2" and "4" involve assessment, but the nurse— not a machine or diagnostic test—should always assess the client.

27. The client is scheduled for a right femoral cardiac catheterization. Which nursing intervention should the nurse implement after the procedure? 1. Perform passive range-of-motion exercises. 2. Assess the client's neurovascular status. 3. Keep the client in high Fowler's position. 4. Assess the gag reflex prior to feeding the client.

2. The nurse must make sure that blood is circulating to the right leg, so the client should be assessed for pulses, paresthesia, paralysis, coldness, and pallor. TEST-TAKING HINT: The nurse should apply the nursing process when determining the correct answer. Therefore, either option "2" or option "4" could possibly be the correct answer. The test taker then should apply anatomy concepts—where is the left femoral artery? Neurovascular assessment is performed on extremities.

36. The charge nurse is making assignments for clients on a cardiac unit. Which client should the charge nurse assign to a new graduate nurse? 1. The 44-year-old client diagnosed with a myocardial infarction. 2. The 65-year-old client admitted with unstable angina. 3. The 75-year-old client scheduled for a cardiac catheterization. 4. The 50-year-old client complaining of chest pain.

3. A new graduate should be able to complete a preprocedure checklist and get this client to the catheterization laboratory. TEST-TAKING HINT: "New graduate" is the key to answering this question correctly. What type of client should be assigned to an inexperienced nurse? The test taker should not assign the new graduate a client who is unstable or at risk

19. The client diagnosed with a myocardial infarction is six (6) hours post-right femoral percutaneous transluminal coronary angioplasty (PTCA), also known as balloon surgery. Which assessment data would require immediate intervention by the nurse? 1. The client is keeping the affected extremity straight. 2. The pressure dressing to the right femoral area is intact. 3. The client is complaining of numbness in the right foot. 4. The client's right pedal pulse is 3+ and bounding.

3. Any neurovascular assessment data that are abnormal require intervention by the nurse; numbness may indicate decreased blood supply to the right foot. TEST-TAKING HINT: This question requires the test taker to identify abnormal, unexpected, or life-threatening data. The nurse must know that a PTCA is performed by placing a catheter in the femoral artery and that internal or external bleeding is the most common complication.

17. The client who has had a myocardial infarction is admitted to the telemetry unit from intensive care. Which referral would be most appropriate for the client? 1. Social worker. 2. Physical therapy. 3. Cardiac rehabilitation. 4. Occupational therapy.

3. Cardiac rehabilitation is the most appropriate referral. The client can start rehabilitation in the hospital and then attend an outpatient cardiac rehabilitation clinic, which includes progressive exercise, diet teaching, and classes on modifying risk factors. TEST-TAKING HINT: The test taker must be familiar with the responsibilities of the other members of the health-care team. If the test taker had no idea which would be the most appropriate referral, the word "cardiac," which means "heart," should help the test taker in deciding that this is the most sensible option because the client had a myocardial infarction, a "heart attack."

24. The client has just returned from a cardiac catheterization. Which assessment data would warrant immediate intervention from the nurse? 1. The client's BP is 110/70 and pulse is 90. 2. The client's groin dressing is dry and intact. 3. The client refuses to keep the leg straight. 4. The client denies any numbness and tingling.

3. If the client bends the leg, it could cause the insertion site to bleed. This is arterial blood and the client could bleed to death very quickly, so this requires immediate intervention. TEST-TAKING HINT: "Warrants immediate intervention" means the nurse should probably notify the health-care provider or do something independently because a complication may occur. Therefore, the test taker must select an answer option that is abnormal or unsafe. In the data listed, there are three normal findings and one abnormal finding.

31. The elderly client has coronary artery disease. Which question should the nurse ask the client during the client teaching? 1. "Do you have a daily bowel movement?" 2. "Do you get yearly chest x-rays (CXRs)?" 3. "Are you sexually active?" 4. "Have you had any weight change?"

3. Sexual activity is a risk factor for angina resulting from coronary artery disease. The client's being elderly should not affect the nurse's assessment of the client's concerns about sexual activity. TEST-TAKING HINT: Remember, if the client is described with an adjective such as "elderly," this may be the key to selecting the correct answer. The nurse must not be judgmental about the elderly, especially about issues concerning sexual activity.

15. The client diagnosed with rule-out myocardial infarction is experiencing chest pain while walking to the bathroom. Which action should the nurse implement first? 1. Administer sublingual nitroglycerin. 2. Obtain a STAT electrocardiogram (ECG). 3. Have the client sit down immediately. 4. Assess the client's vital signs.

3. Stopping all activity will decrease the need of the myocardium for oxygen and may help decrease the chest pain. TEST-TAKING HINT: Whenever the test taker wants to select an assessment intervention, be sure to think about whether that intervention will help the client, especially if the client is experiencing pain. Do not automatically select the answer option that is assessment.

22. The client diagnosed with a myocardial infarction is on bedrest. The unlicensed assistive personnel (UAP) is encouraging the client to move the legs. Which action should the nurse implement? 1. Instruct the UAP to stop encouraging the leg movements. 2. Report this behavior to the charge nurse as soon as possible. 3. Praise the UAP for encouraging the client to move the legs. 4. Take no action concerning the UAP's behavior

3. The nurse should praise and encourage UAPs to participate in the client's care. Clients on bedrest are at risk for deep vein thrombosis, and moving the legs will help prevent this from occurring. TEST-TAKING HINT: This is a management question. The test taker must know the chain of command and when to report behavior. The test taker could eliminate options "1" and "2" with the knowledge that moving the legs is a safe activity for the client. When having to choose between options "3" and "4," the test taker should select doing something positive, instead of taking no action. This is a management concept.

13. Which cardiac enzyme would the nurse expect to elevate first in a client diagnosed with a myocardial infarction (MI)? 1. Creatine kinase (CK-MB). 2. Lactate dehydrogenase (LDH). 3. Troponin. 4. White blood cells (WBCs).

3. Troponin is the enzyme that elevates within 1 to 2 hours. TEST-TAKING HINT: The test taker should be aware of the words "cardiac enzyme," which would eliminate option "4" as a possible answer. The word in the stem is "first." This questionrequires the test taker to have knowledge of laboratory values.

32. The nurse is discussing the importance of exercise with the client diagnosed with coronary artery disease. Which intervention should the nurse implement? 1. Perform isometric exercises daily. 2. Walk for 15 minutes three (3) times a week. 3. Do not walk outside if it is less than 40˚F. 4. Wear open-toed shoes when ambulating.

3. When it is cold outside, vasoconstriction occurs, and this will decrease oxygen to the heart muscle. Therefore, the client should not exercise when it is cold outside. TEST-TAKING HINT: The test taker should be aware of adjectives such as "isometric," which makes option "1" incorrect, and "open-toed," which makes option "4" incorrect.

COMPLETION 1. A nurse prepares a client with acute renal insufficiency for a cardiac catheterization. The provider prescribes 0.9% normal saline to infuse at 125 mL/hr for renal protection. The nurse obtains gravity tubing with a drip rate of 15 drops/mL. At what rate (drops/min) should the nurse infuse the fluids? (Record your answer using a whole number, and rounding to the nearest drop.) drops/min

31 drops/min

25. The male client is diagnosed with coronary artery disease (CAD) and is prescribed sublingual nitroglycerin. Which statement indicates the client needs more teaching? 1. "I should keep the tablets in the dark-colored bottle they came in." 2. "If the tablets do not burn under my tongue, they are not effective." 3. "I should keep the bottle with me in my pocket at all times." 4. "If my chest pain is not

4. The client should take one tablet every five (5) minutes and, if no relief occurs after the third tablet, have someone drive him to the emergency department or call 911. TEST-TAKING HINT: This question is an "except" question, requiring the test taker to identify which statement indicates the client doesn't understand the teaching. Sometimes the test taker could restate the question and think which statement indicates the client understands the teaching.

21. The nurse is administering a calcium channel blocker to the client diagnosed with a myocardial infarction. Which assessment data would cause the nurse to question administering this medication? 1. The client's apical pulse is 64. 2. The client's calcium level is elevated. 3. The client's telemetry shows occasional PVCs. 4. The client's blood pressure is 90/58.

4. The client's blood pressure is low, and a calcium channel blocker could cause the blood pressure to bottom out. TEST-TAKING HINT: The test taker must know when to question administering medications. The test taker is trying to select an option that, if the medication is administered, would cause serious harm to the client.

a client weighing 174 lbs had thrombolytic therapy followed by a one-time dose of IV lovenox 30 mg. the physician prescribes lovenox 1 mg/kg subcutaneously after the IV administration. the nurse will give ____ mg of lovenox to the client.

79

5. An emergency room nurse obtains the health history of a client. Which statement by the client should alert the nurse to the occurrence of heart failure? a. I get short of breath when I climb stairs. b. I see halos floating around my head. c. I have trouble remembering things. d. I have lost weight over the past month.

A Dyspnea on exertion is an early manifestation of heart failure and is associated with an activity such as stair climbing. The other findings are not specific to early occurrence of heart failure.

12. A nurse assesses a client who is recovering from a myocardial infarction. The clients pulmonary artery pressure reading is 25/12 mm Hg. Which action should the nurse take first? a. Compare the results with previous pulmonary artery pressure readings. b. Increase the intravenous fluid rate because these readings are low. c. Immediately notify the health care provider of the elevated pressures. d. Document the finding in the clients chart as the only action.

A Normal pulmonary artery pressures range from 15 to 26 mm Hg for systolic and from 5 to 15 mm Hg for diastolic. Although this clients readings are within normal limits, the nurse needs to assess any trends that may indicate a need for medical treatment to prevent complications. There is no need to increase intravenous fluids or notify the provider.

21. A nurse assesses a client who has aortic regurgitation. In which location in the illustration shown below should the nurse auscultate to best hear a cardiac murmur related to aortic regurgitation? a. Location A b. Location B c. Location C d. Location D

A The aortic valve is auscultated in the second intercostal space just to the right of the sternum.

1. A nurse assesses a client who had a myocardial infarction and is hypotensive. Which additional assessment finding should the nurse expect? a. Heart rate of 120 beats/min b. Cool, clammy skin c. Oxygen saturation of 90% d. Respiratory rate of 8 breaths/min

A When a client experiences hypotension, baroreceptors in the aortic arch sense a pressure decrease in the vessels. The parasympathetic system responds by lessening the inhibitory effect on the sinoatrial node. This results in an increase in heart rate and respiratory rate. This tachycardia is an early response and is seen even when blood pressure is not critically low. An increased heart rate and respiratory rate will compensate for the low blood pressure and maintain oxygen saturations and perfusion. The client may not be able to compensate for long, and decreased oxygenation and cool, clammy skin will occur later.

An LPN/LVN is scheduled to work on the inpatient "stepdown" cardiac unit. Which client does the charge nurse assign to the LPN/LVN? A 60-year-old who was admitted today for pacemaker insertion because of third-degree heart block and who is now reporting chest pain A 62-year-old who underwent open heart surgery 4 days ago for mitral valve replacement and who has a temperature of 38.2° C A 66-year-old who has a prescription for a nitroglycerin (Nitro-Dur) patch and is scheduled for discharge to a group home later today A 69-year-old who had a stent placed 2 hours ago in the left anterior descending artery and who has bursts of ventricular tachycardia

A 66-year-old who has a prescription for a nitroglycerin (Nitro-Dur) patch and is scheduled for discharge to a group home later today The LPN/LVN scope of practice includes administration of medications to stable clients. Third-degree heart block is characterized by a very low heart rate and usually by required pacemaker insertion; the skills of the RN are needed to care for this client. Fever after surgery requires collaboration with the health care provider, which is more consistent with the role of the RN. The client with a recent stent placement and having bursts of ventricular tachycardia is unstable and is showing ventricular irritability; he will need medications and monitoring beyond the scope of practice of the LPN/LVN.

1. A nurse is caring for a client with a history of renal insufficiency who is scheduled for a cardiac catheterization. Which actions should the nurse take prior to the catheterization? (Select all that apply.) a. Assess for allergies to iodine. b. Administer intravenous fluids. c. Assess blood urea nitrogen (BUN) and creatinine results. d. Insert a Foley catheter. e. Administer a prophylactic antibiotic. f. Insert a central venous catheter.

A, B, C If the client has kidney disease (as indicated by BUN and creatinine results), fluids and Mucomyst may be given 12 to 24 hours before the procedure for renal protection. The client should be assessed for allergies to iodine, including shellfish; the contrast medium used during the catheterization contains iodine. A Foley catheter and central venous catheter are not required for the procedure and would only increase the clients risk for infection. Prophylactic antibiotics are not administered prior to a cardiac catheterization.

4. A nurse reviews a clients laboratory results. Which findings should alert the nurse to the possibility of atherosclerosis? (Select all that apply.) a. Total cholesterol: 280 mg/dL b. High-density lipoprotein cholesterol: 50 mg/dL c. Triglycerides: 200 mg/dL d. Serum albumin: 4 g/dL e. Low-density lipoprotein cholesterol: 160 mg/dL

A, C, E A lipid panel is often used to screen for cardiovascular risk. Total cholesterol, triglycerides, and low-density lipoprotein cholesterol levels are all high, indicating higher risk for cardiovascular disease. High-density lipoprotein cholesterol is within the normal range for both males and females. Serum albumin is not assessed for atherosclerosis.

6. A nurse cares for a client who is recovering from a right-sided heart catheterization. For which complications of this procedure should the nurse assess? (Select all that apply.) a. Thrombophlebitis b. Stroke c. Pulmonary embolism d. Myocardial infarction e. Cardiac tamponade

A, C, E Complications from a right-sided heart catheterization include thrombophlebitis, pulmonary embolism, and vagal response. Cardiac tamponade is a risk of both right- and left-sided heart catheterizations. Stroke and myocardial infarction are complications of left-sided heart catheterizations.

21. A client is on a dopamine infusion via a peripheral line. What action by the nurse takes priority for safety? a. Assess the IV site hourly. b. Monitor the pedal pulses. c. Monitor the client's vital signs. d. Obtain consent for a central line.

ANS: A Dopamine should be infused through a central line to prevent extravasation and necrosis of tissue. If it needs to be run peripherally, the nurse assesses the site hourly for problems. When the client is getting the central line, ensuring informed consent is on the chart is a priority. But at this point, the client has only a peripheral line, so caution must be taken to preserve the integrity of the client's integumentary system. Monitoring pedal pulses and vital signs give indications as to how well the drug is working. DIF: Applying/Application REF: 773 KEY: Inotropic agents| adverse effects| medication safety MSC: IntegratedProcess:NursingProcess:Assessment NOT: Client Needs Category: Physiological Integrity: Pharmacological and Parenteral Therapies

18. A nurse is caring for a client who is intubated and has an intra-aortic balloon pump. The client is restless and agitated. What action should the nurse perform first for comfort? a. Allow family members to remain at the bedside. b. Ask the family if the client would like a fan in the room. c. Keep the television tuned to the client's favorite channel. d. Speak loudly to the client in case of hearing problems.

ANS: A Allowing the family to remain at the bedside can help calm the client with familiar voices (and faces if the client wakes up). A fan might be helpful but may also spread germs through air movement. The TV should not be kept on all the time to allow for rest. Speaking loudly may agitate the client more. DIF: Applying/Application REF: 780 KEY: Intra-aortic balloon pump| nonpharmacologic comfort measures MSC: Integrated Process: Nursing Process: Implementation NOT: Client Needs Category: Physiological Integrity: Basic Care and Comfort

1. A nurse assesses clients on a cardiac unit. Which client should the nurse identify as being at greatest risk for the development of left-sided heart failure? a. A 36-year-old woman with aortic stenosis b. A 42-year-old man with pulmonary hypertension c. A 59-year-old woman who smokes cigarettes daily d. A 70-year-old man who had a cerebral vascular accident

ANS: A Although most people with heart failure will have failure that progresses from left to right, it is possible to have left-sided failure alone for a short period. It is also possible to have heart failure that progresses from right to left. Causes of left ventricular failure include mitral or aortic valve disease, coronary artery disease, and hypertension. Pulmonary hypertension and chronic cigarette smoking are risk factors for right ventricular failure. A cerebral vascular accident does not increase the risk of heart failure. DIF: Applying/Application REF: 679 KEY: Heart failure| health screening MSC: IntegratedProcess:NursingProcess:Assessment NOT: Client Needs Category: Safe and Effective Care Environment: Management of Care

6. A nurse is teaching a client with heart failure who has been prescribed enalapril (Vasotec). Which statement should the nurse include in this client's teaching? a. "Avoid using salt substitutes." b. "Take your medication with food." c. "Avoid using aspirin-containing products." d. "Check your pulse daily."

ANS: A Angiotensin-converting enzyme (ACE) inhibitors such as enalapril inhibit the excretion of potassium. Hyperkalemia can be a life-threatening side effect, and clients should be taught to limit potassium intake. Salt substitutes are composed of potassium chloride. ACE inhibitors do not need to be taken with food and have no impact on the client's pulse rate. Aspirin is often prescribed in conjunction with ACE inhibitors and is not contraindicated. DIF: Applying/Application REF: 685 KEY: Heart failure| angiotensin-converting enzyme (ACE) inhibitor| medication| patient education MSC: IntegratedProcess:Teaching/Learning NOT: Client Needs Category: Physiological Integrity: Pharmacological and Parenteral Therapies

11. A nurse admits a client who is experiencing an exacerbation of heart failure. Which action should the nurse take first? a. Assess the client's respiratory status. b. Draw blood to assess the client's serum electrolytes. c. Administer intravenous furosemide (Lasix). d. Ask the client about current medications.

ANS: A Assessment of respiratory and oxygenation status is the priority nursing intervention for the prevention of complications. Monitoring electrolytes, administering diuretics, and asking about current medications are important but do not take priority over assessing respiratory status. DIF: Applying/Application REF: 687 KEY: Heart failure| respiratory distress/failure| assessment/diagnostic examination MSC: Integrated Process: Nursing Process: Implementation NOT: Client Needs Category: Safe and Effective Care Environment: Management of Care

14. A nurse assesses a client with tachycardia. Which clinical manifestation requires immediate intervention by the nurse? a. Mid-sternal chest pain b. Increased urine output c. Mild orthostatic hypotension d. P wave touching the T wave

ANS: A Chest pain, possibly angina, indicates that tachycardia may be increasing the client's myocardial workload and oxygen demand to such an extent that normal oxygen delivery cannot keep pace. This results in myocardial hypoxia and pain. Increased urinary output and mild orthostatic hypotension are not life-threatening conditions and therefore do not require immediate intervention. The P wave touching the T wave indicates significant tachycardia and should be assessed to determine the underlying rhythm and cause; this is an important assessment but is not as critical as chest pain, which indicates cardiac cell death. DIF: Applying/Application REF: 663 KEY: Cardiac electrical conduction MSC: IntegratedProcess:NursingProcess:Assessment NOT: Client Needs Category: Safe and Effective Care Environment: Management of Care

4. A nursing student is caring for a client who had a myocardial infarction. The student is confused because the client states nothing is wrong and yet listens attentively while the student provides education on lifestyle changes and healthy menu choices. What response by the faculty member is best? a. "Continue to educate the client on possible healthy changes." b. "Emphasize complications that can occur with noncompliance." c. "Tell the client that denial is normal and will soon go away." d. "You need to make sure the client understands this illness."

ANS: A Clients are often in denial after a coronary event. The client who seems to be in denial but is compliant with treatment may be using a healthy form of coping that allows time to process the event and start to use problem-focused coping. The student should not discourage this type of denial and coping, but rather continue providing education in a positive manner. Emphasizing complications may make the client defensive and more anxious. Telling the client that denial is normal is placing too much attention on the process. Forcing the client to verbalize understanding of the illness is also potentially threatening to the client. DIF: Understanding/Comprehension REF: 769 KEY: Coronary artery disease| psychosocial response| coping| therapeutic communication MSC: Integrated Process: Communication and Documentation NOT: Client Needs Category: Psychosocial Integrity

26. A client had an inferior wall myocardial infarction (MI). The nurse notes the client's cardiac rhythm as shown below: What action by the nurse is most important? a. Assess the client's blood pressure and level of consciousness. b. Call the health care provider or the Rapid Response Team. c. Obtain a permit for an emergency temporary pacemaker insertion. d. Prepare to administer antidysrhythmic medication.

ANS: A Clients with an inferior wall MI often have bradycardia and blocks that lead to decreased perfusion, as seen in this ECG strip showing sinus bradycardia. The nurse should first assess the client's hemodynamic status, including vital signs and level of consciousness. The client may or may not need the Rapid Response Team, a temporary pacemaker, or medication; there is no indication of this in the question. DIF: Analyzing/Analysis REF: 769 KEY: Coronary artery disease| dysrhythmias| nursing assessment| hemodynamic status MSC: Integrated Process: Nursing Process: Implementation NOT: Client Needs Category: Physiological Integrity: Physiological Adaptation

9. A client is in the clinic a month after having a myocardial infarction. The client reports sleeping well since moving into the guest bedroom. What response by the nurse is best? a. "Do you have any concerns about sexuality?" b. "I'm glad to hear you are sleeping well now." c. "Sleep near your spouse in case of emergency." d. "Why would you move into the guest room?"

ANS: A Concerns about resuming sexual activity are common after cardiac events. The nurse should gently inquire if this is the issue. While it is good that the client is sleeping well, the nurse should investigate the reason for the move. The other two responses are likely to cause the client to be defensive. DIF: Applying/Application REF: 781 KEY: Coronary artery disease| sexuality| anxiety| therapeutic communication MSC: IntegratedProcess:Caring NOT: Client Needs Category: Psychosocial Integrity

5. A nurse cares for a client with right-sided heart failure. The client asks, "Why do I need to weigh myself every day?" How should the nurse respond? a. "Weight is the best indication that you are gaining or losing fluid." b. "Daily weights will help us make sure that you're eating properly." c. "The hospital requires that all inpatients be weighed daily." d. "You need to lose weight to decrease the incidence of heart failure."

ANS: A Daily weights are needed to document fluid retention or fluid loss. One liter of fluid equals 2.2 pounds. The other responses do not address the importance of monitoring fluid retention or loss. DIF: Remembering/Knowledge REF: 683 KEY: Heart failure| patient education MSC: Integrated Process: Teaching/Learning NOT: Client Needs Category: Physiological Integrity: Physiological Adaptation

21. A nurse cares for an older adult client with heart failure. The client states, "I don't know what to do. I don't want to be a burden to my daughter, but I can't do it alone. Maybe I should die." How should the nurse respond? a. "Would you like to talk more about this?" b. "You are lucky to have such a devoted daughter." c. "It is normal to feel as though you are a burden." d. "Would you like to meet with the chaplain?"

ANS: A Depression can occur in clients with heart failure, especially older adults. Having the client talk about his or her feelings will help the nurse focus on the actual problem. Open-ended statements allow the client to respond safely and honestly. The other options minimize the client's concerns and do not allow the nurse to obtain more information to provide client-centered care. DIF: Applying/Application REF: 683 KEY: Heart failure| support| psychosocial response MSC: IntegratedProcess:Caring NOT: Client Needs Category: Psychosocial Integrity

11. An older adult is on cardiac monitoring after a myocardial infarction. The client shows frequent dysrhythmias. What action by the nurse is most appropriate? a. Assess for any hemodynamic effects of the rhythm. b. Prepare to administer antidysrhythmic medication. c. Notify the provider or call the Rapid Response Team. d. Turn the alarms off on the cardiac monitor.

ANS: A Older clients may have dysrhythmias due to age-related changes in the cardiac conduction system. They may have no significant hemodynamic effects from these changes. The nurse should first assess for the effects of the dysrhythmia before proceeding further. The alarms on a cardiac monitor should never be shut off. The other two actions may or may not be needed. DIF: Applying/Application REF: 769 KEY: Coronary artery disease| older adult| pathophysiology| nursing assessment MSC: IntegratedProcess:NursingProcess:Assessment NOT: Client Needs Category: Health Promotion and Maintenance

15. A nurse teaches a client who experiences occasional premature atrial contractions (PACs) accompanied by palpitations that resolve spontaneously without treatment. Which statement should the nurse include in this client's teaching? a. "Minimize or abstain from caffeine." b. "Lie on your side until the attack subsides." c. "Use your oxygen when you experience PACs." d. "Take amiodarone (Cordarone) daily to prevent PACs."

ANS: A PACs usually have no hemodynamic consequences. For a client experiencing infrequent PACs, the nurse should explore possible lifestyle causes, such as excessive caffeine intake and stress. Lying on the side will not prevent or resolve PACs. Oxygen is not necessary. Although medications may be needed to control symptomatic dysrhythmias, for infrequent PACs, the client first should try lifestyle changes to control them. DIF: Applying/Application REF: 663 KEY: Patient education| cardiac electrical conduction MSC: IntegratedProcess:Teaching/Learning NOT: Client Needs Category: Health Promotion and Maintenance

15. A nurse cares for a client with infective endocarditis. Which infection control precautions should the nurse use? a. Standard Precautions b. Bleeding precautions c. Reverse isolation d. Contact isolation

ANS: A The client with infective endocarditis does not pose any specific threat of transmitting the causative organism. Standard Precautions should be used. Bleeding precautions or reverse or contact isolation is not necessary. DIF: Applying/Application REF: 697 KEY: Infection| Standard Precautions MSC: Integrated Process: Nursing Process: Implementation NOT: Client Needs Category: Safe and Effective Care Environment: Safety and Infection Control

13. A nurse prepares to discharge a client with cardiac dysrhythmia who is prescribed home health care services. Which priority information should be communicated to the home health nurse upon discharge? a. Medication reconciliation b. Immunization history c. Religious beliefs d. Nutrition preferences

ANS: A The home health nurse needs to know current medications the client is taking to ensure assessment, evaluation, and further education related to these medications. The other information will not assist the nurse to develop a plan of care for the client. DIF: Applying/Application REF: 673 KEY: Hand-off communication MSC: Integrated Process: Communication and Documentation NOT: Client Needs Category: Safe and Effective Care Environment: Management of Care

4. While assessing a client on a cardiac unit, a nurse identifies the presence of an S3 gallop. Which action should the nurse take next? a. Assess for symptoms of left-sided heart failure. b. Document this as a normal finding. c. Call the health care provider immediately. d. Transfer the client to the intensive care unit.

ANS: A The presence of an S3 gallop is an early diastolic filling sound indicative of increasing left ventricular pressure and left ventricular failure. The other actions are not warranted. DIF: Remembering/Knowledge REF: 683 KEY: Heart failure| assessment/diagnostic examination MSC: IntegratedProcess:NursingProcess:Assessment NOT: Client Needs Category: Physiological Integrity: Reduction of Risk Potential

17. A nurse supervises an unlicensed assistive personnel (UAP) applying electrocardiographic monitoring. Which statement should the nurse provide to the UAP related to this procedure? a. "Clean the skin and clip hairs if needed." b. "Add gel to the electrodes prior to applying them." c. "Place the electrodes on the posterior chest." d. "Turn off oxygen prior to monitoring the client."

ANS: A To ensure the best signal transmission, the skin should be clean and hairs clipped. Electrodes should be placed on the anterior chest, and no additional gel is needed. Oxygen has no impact on electrocardiographic monitoring. DIF: Remembering/Knowledge REF: 652 KEY: Assessment/diagnostic examination| interdisciplinary team| unlicensed assistive personnel (UAP) MSC: Integrated Process: Communication and Documentation NOT: Client Needs Category: Safe and Effective Care Environment: Management of Care

19. A nurse cares for a client who is on a cardiac monitor. The monitor displayed the rhythm shown below: Which action should the nurse take first? a. Assess airway, breathing, and level of consciousness. b. Administer an amiodarone bolus followed by a drip. c. Cardiovert the client with a biphasic defibrillator. d. Begin cardiopulmonary resuscitation (CPR).

ANS: A Ventricular tachycardia occurs with repetitive firing of an irritable ventricular ectopic focus, usually at a rate of 140 to 180 beats/min or more. Ventricular tachycardia is a lethal dysrhythmia. The nurse should first assess if the client is alert and breathing. Then the nurse should call a Code Blue and begin CPR. If this client is pulseless, the treatment of choice is defibrillation. Amiodarone is the antidysrhythmic of choice, but it is not the first action. DIF: Applying/Application REF: 670 KEY: Cardiac electrical conduction| medical emergency MSC: Integrated Process: Nursing Process: Implementation NOT: Client Needs Category: Physiological Integrity: Physiological Adaptation

8. A nurse assesses a client who is recovering from a heart transplant. Which assessment findings should alert the nurse to the possibility of heart transplant rejection? (Select all that apply.) a. Shortness of breath b. Abdominal bloating c. New-onset bradycardia d. Increased ejection fraction e. Hypertension

ANS: A, B, C Clinical manifestations of heart transplant rejection include shortness of breath, fatigue, fluid gain, abdominal bloating, new-onset bradycardia, hypotension, atrial fibrillation or flutter, decreased activity tolerance, and decreased ejection fraction. DIF: Remembering/Knowledge REF: 703 KEY: Transplant| heart failure MSC: IntegratedProcess:NursingProcess:Assessment NOT: Client Needs Category: Physiological Integrity: Reduction of Risk Potential

5. A nursing student planning to teach clients about risk factors for coronary artery disease (CAD) would include which topics? (Select all that apply.) a. Advanced age b. Diabetes c. Ethnic background d. Medication use e. Smoking

ANS: A, B, C, E Age, diabetes, ethnic background, and smoking are all risk factors for developing CAD; medication use is not. DIF: Remembering/Knowledge REF: 760 KEY: Coronary artery disease| pathophysiology| patient education MSC: IntegratedProcess:Teaching/Learning NOT: Client Needs Category: Physiological Integrity: Physiological Adaptation

3. A nurse is teaching a client with premature ectopic beats. Which education should the nurse include in this client's teaching? (Select all that apply.) a. Smoking cessation b. Stress reduction and management c. Avoiding vagal stimulation d. Adverse effects of medications e. Foods high in potassium

ANS: A, B, D A client who has premature beats or ectopic rhythms should be taught to stop smoking, manage stress, take medications as prescribed, and report adverse effects of medications. Clients with premature beats are not at risk for vasovagal attacks or potassium imbalances. DIF: Remembering/Knowledge REF: 673 KEY: Patient education MSC: IntegratedProcess:NursingProcess:Implementation NOT: Client Needs Category: Health Promotion and Maintenance

3. A nurse assesses clients on a cardiac unit. Which clients should the nurse identify as at greatest risk for the development of acute pericarditis? (Select all that apply.) a. A 36-year-old woman with systemic lupus erythematosus (SLE) b. A 42-year-old man recovering from coronary artery bypass graft surgery c. A 59-year-old woman recovering from a hysterectomy d. An 80-year-old man with a bacterial infection of the respiratory tract e. An 88-year-old woman with a stage III sacral ulcer

ANS: A, B, D Acute pericarditis is most commonly associated acute exacerbations of systemic connective tissue disease, including SLE; with Dressler's syndrome, or inflammation of the cardiac sac after cardiac surgery or a myocardial infarction; and with infective organisms, including bacterial, viral, and fungal infections. Abdominal and reproductive surgeries and pressure ulcers do not increase clients' risk for acute pericarditis. DIF: Applying/Application REF: 699 KEY: Inflammatory response| health screening MSC: IntegratedProcess:NursingProcess:Assessment NOT: Client Needs Category: Safe and Effective Care Environment: Management of Care

6. A nurse prepares to discharge a client who has heart failure. Based on the Heart Failure Core Measure Set, which actions should the nurse complete prior to discharging this client? (Select all that apply.) a. Teach the client about dietary restrictions. b. Ensure the client is prescribed an angiotensin-converting enzyme (ACE) inhibitor. c. Encourage the client to take a baby aspirin each day. d. Confirm that an echocardiogram has been completed. e. Consult a social worker for additional resources.

ANS: A, B, D The Heart Failure Core Measure Set includes discharge instructions on diet, activity, medications, weight monitoring and plan for worsening symptoms, evaluation of left ventricular systolic function (usually with an echocardiogram), and prescribing an ACE inhibitor or angiotensin receptor blocker. Aspirin is not part of the Heart Failure Core Measure Set and is usually prescribed for clients who experience a myocardial infarction. Although the nurse may consult the social worker or case manager for additional resources, this is not part of the Core Measures. DIF: Understanding/Comprehension REF: 689 KEY: Heart failure| discharge| Core Measures| The Joint Commission MSC: Integrated Process: Nursing Process: Analysis NOT: Client Needs Category: Safe and Effective Care Environment: Management of Care

7. A nurse prepares to discharge a client who has heart failure. Which questions should the nurse ask to ensure this client's safety prior to discharging home? (Select all that apply.) a. "Are your bedroom and bathroom on the first floor?" b. "What social support do you have at home?" c. "Will you be able to afford your oxygen therapy?" d. "What spiritual beliefs may impact your recovery?" e. "Are you able to accurately weigh yourself at home?"

ANS: A, B, D To ensure safety upon discharge, the nurse should assess for structural barriers to functional ability, such as stairs. The nurse should also assess the client's available social support, which may include family, friends, and home health services. The client's ability to adhere to medication and treatments, including daily weights, should also be reviewed. The other questions do not address the client's safety upon discharge. DIF: Applying/Application REF: 689 KEY: Heart failure| discharge| safety MSC: Integrated Process: Nursing Process: Analysis NOT: Client Needs Category: Safe and Effective Care Environment: Safety and Infection Control

3. A nursing student studying acute coronary syndromes learns that the pain of a myocardial infarction (MI) differs from stable angina in what ways? (Select all that apply.) a. Accompanied by shortness of breath b. Feelings of fear or anxiety c. Lasts less than 15 minutes d. No relief from taking nitroglycerin e. Pain occurs without known cause

ANS: A, B, D, E The pain from an MI is often accompanied by shortness of breath and fear or anxiety. It lasts longer than 15 minutes and is not relieved by nitroglycerin. It occurs without a known cause such as exertion. DIF: Remembering/Knowledge REF: 762 KEY: Coronary artery disease| pathophysiology MSC: IntegratedProcess:Teaching/Learning NOT: Client Needs Category: Physiological Integrity: Physiological Adaptation

MULTIPLE RESPONSE 1. A nurse is assessing a client with left-sided heart failure. For which clinical manifestations should the nurse assess? (Select all that apply.) a. Pulmonary crackles b. Confusion, restlessness c. Pulmonary hypertension d. Dependent edema e. Cough that worsens at night

ANS: A, B, E Left-sided heart failure occurs with a decrease in contractility of the heart or an increase in afterload. Most of the signs will be noted in the respiratory system. Right-sided heart failure occurs with problems from the pulmonary vasculature onward including pulmonary hypertension. Signs will be noted before the right atrium or ventricle including dependent edema. DIF: Remembering/Knowledge REF: 682 KEY: Heart failure| assessment/diagnostic examination MSC: IntegratedProcess:NursingProcess:Assessment NOT: Client Needs Category: Physiological Integrity: Physiological Adaptation

2. A nurse teaches a client with a new permanent pacemaker. Which instructions should the nurse include in this client's teaching? (Select all that apply.) a. "Until your incision is healed, do not submerge your pacemaker. Only take showers." b. "Report any pulse rates lower than your pacemaker settings." c. "If you feel weak, apply pressure over your generator." d. "Have your pacemaker turned off before having magnetic resonance imaging (MRI)." e. "Do not lift your left arm above the level of your shoulder for 8 weeks."

ANS: A, B, E The client should not submerge in water until the site has healed; after the incision is healed, the client may take showers or baths without concern for the pacemaker. The client should be instructed to report changes in heart rate or rhythm, such as rates lower than the pacemaker setting or greater than 100 beats/min. The client should be advised of restrictions on physical activity for 8 weeks to allow the pacemaker to settle in place. The client should never apply pressure over the generator and should avoid tight clothing. The client should never have MRI because, whether turned on or off, the pacemaker contains metal. The client should be advised to inform all health care providers that he or she has a pacemaker. DIF: Applying/Application REF: 675 KEY: Cardiac electrical conduction| patient education MSC: IntegratedProcess:Teaching/Learning NOT: Client Needs Category: Health Promotion and Maintenance

2. A nurse evaluates laboratory results for a client with heart failure. Which results should the nurse expect? (Select all that apply.) a. Hematocrit: 32.8% b. Serum sodium: 130 mEq/L c. Serum potassium: 4.0 mEq/L d. Serum creatinine: 1.0 mg/dL e. Proteinuria f. Microalbuminuria

ANS: A, B, E, F A hematocrit of 32.8% is low (should be 42.6%), indicating a dilutional ratio of red blood cells to fluid. A serum sodium of 130 mEq/L is low because of hemodilution. Microalbuminuria and proteinuria are present, indicating a decrease in renal filtration. These are early warning signs of decreased compliance of the heart. The potassium level is on the high side of normal and the serum creatinine level is normal. DIF: Applying/Application REF: 683 KEY: Heart failure| assessment/diagnostic examination MSC: Integrated Process: Nursing Process: Evaluation NOT: Client Needs Category: Physiological Integrity: Reduction of Risk Potential

5. A nurse collaborates with an unlicensed assistive personnel (UAP) to provide care for a client with congestive heart failure. Which instructions should the nurse provide to the UAP when delegating care for this client? (Select all that apply.) a. "Reposition the client every 2 hours." b. "Teach the client to perform deep-breathing exercises." c. "Accurately record intake and output." d. "Use the same scale to weigh the client each morning." e. "Place the client on oxygen if the client becomes short of breath."

ANS: A, C, D The UAP should reposition the client every 2 hours to improve oxygenation and prevent atelectasis. The UAP can also accurately record intake and output, and use the same scale to weigh the client each morning before breakfast. UAPs are not qualified to teach clients or assess the need for and provide oxygen therapy. DIF: Applying/Application REF: 684 KEY: Heart failure| delegation| interdisciplinary team| unlicensed assistive personnel (UAP) MSC: Integrated Process: Communication and Documentation NOT: Client Needs Category: Safe and Effective Care Environment: Management of Care

2. A nurse is caring for a client who had coronary artery bypass grafting yesterday. What actions does the nurse delegate to the unlicensed assistive personnel (UAP)? (Select all that apply.) a. Assist the client to the chair for meals and to the bathroom. b. Encourage the client to use the spirometer every 4 hours. c. Ensure the client wears TED hose or sequential compression devices. d. Have the client rate pain on a 0-to-10 scale and report to the nurse. e. Take and record a full set of vital signs per hospital protocol.

ANS: A, C, E The nurse can delegate assisting the client to get up in the chair or ambulate to the bathroom, applying TEDs or sequential compression devices, and taking/recording vital signs. The spirometer should be used every hour the day after surgery. Assessing pain using a 0-to-10 scale is a nursing assessment, although if the client reports pain, the UAP should inform the nurse so a more detailed assessment is done. DIF: Applying/Application REF: 768 KEY: Coronary artery disease| coronary artery bypass graft| delegation| activity| unlicensed assistive personnel (UAP) MSC: Integrated Process: Communication and Documentation NOT: Client Needs Category: Safe and Effective Care Environment: Management of Care

4. After teaching a client with congestive heart failure (CHF), the nurse assesses the client's understanding. Which client statements indicate a correct understanding of the teaching related to nutritional intake? (Select all that apply.) a. "I'll read the nutritional labels on food items for salt content." b. "I will drink at least 3 liters of water each day." c. "Using salt in moderation will reduce the workload of my heart." d. "I will eat oatmeal for breakfast instead of ham and eggs." e. "Substituting fresh vegetables for canned ones will lower my salt intake."

ANS: A, D, E Nutritional therapy for a client with CHF is focused on decreasing sodium and water retention to decrease the workload of the heart. The client should be taught to read nutritional labels on all food items, omit table salt and foods high in sodium (e.g., ham and canned foods), and limit water intake to a normal 2 L/day. DIF: Applying/Application REF: 682 KEY: Heart failure| patient education MSC: Integrated Process: Teaching/Learning NOT: Client Needs Category: Physiological Integrity: Basic Care and Comfort

MULTIPLE RESPONSE 1. A nurse cares for a client with congestive heart failure who has a regular cardiac rhythm of 128 beats/min. For which physiologic alterations should the nurse assess? (Select all that apply.) a. Decrease in cardiac output b. Increase in cardiac output c. Decrease in blood pressure d. Increase in blood pressure e. Decrease in urine output f. Increase in urine output

ANS: A, D, E Elevated heart rates in a healthy client initially cause blood pressure and cardiac output to increase. However, in a client who has congestive heart failure or a client with long-term tachycardia, ventricular filling time, cardiac output, and blood pressure eventually decrease. As cardiac output and blood pressure decrease, urine output will fall. DIF: Applying/Application REF: 657 KEY: Cardiac electrical conduction| heart failure MSC: IntegratedProcess:NursingProcess:Assessment NOT: Client Needs Category: Physiological Integrity: Physiological Adaptation

3. A nurse is assessing clients on a medical-surgical unit. Which client should the nurse identify as being at greatest risk for atrial fibrillation? a. A 45-year-old who takes an aspirin daily b. A 50-year-old who is post coronary artery bypass graft surgery c. A 78-year-old who had a carotid endarterectomy d. An 80-year-old with chronic obstructive pulmonary disease

ANS: B Atrial fibrillation occurs commonly in clients with cardiac disease and is a common occurrence after coronary artery bypass graft surgery. The other conditions do not place these clients at higher risk for atrial fibrillation. DIF: Applying/Application REF: 666 KEY: Health screening| cardiac electrical conduction MSC: IntegratedProcess:NursingProcess:Assessment NOT: Client Needs Category: Safe and Effective Care Environment: Management of Care

21. The nurse is caring for a client on the medical-surgical unit who suddenly becomes unresponsive and has no pulse. The cardiac monitor shows the rhythm below: After calling for assistance and a defibrillator, which action should the nurse take next? a. Perform a pericardial thump. b. Initiate cardiopulmonary resuscitation (CPR). c. Start an 18-gauge intravenous line. d. Ask the client's family about code status.

ANS: B The client's rhythm is ventricular fibrillation. This is a lethal rhythm that is best treated with immediate defibrillation. While the nurse is waiting for the defibrillator to arrive, the nurse should start CPR. A pericardial thump is not a treatment for ventricular fibrillation. If the client does not already have an IV, other members of the team can insert one after defibrillation. The client's code status should already be known by the nurse prior to this event. DIF: Applying/Application REF: 671 KEY: Cardiac electrical conduction| medical emergency MSC: IntegratedProcess:NursingProcess:Assessment NOT: Client Needs Category: Physiological Integrity: Physiological Adaptation

22. A nurse teaches a client with heart failure about energy conservation. Which statement should the nurse include in this client's teaching? a. "Walk until you become short of breath, and then walk back home." b. "Gather everything you need for a chore before you begin." c. "Pull rather than push or carry items heavier than 5 pounds." d. "Take a walk after dinner every day to build up your strength."

ANS: B A client who has heart failure should be taught to conserve energy. Gathering all supplies needed for a chore at one time decreases the amount of energy needed. The client should not walk until becoming short of breath because he or she may not make it back home. Pushing a cart takes less energy than pulling or lifting. Although walking after dinner may help the client, the nurse should teach the client to complete activities when he or she has the most energy. This is usually in the morning. DIF: Applying/Application REF: 696 KEY: Heart failure| functional ability| patient education MSC: IntegratedProcess:Teaching/Learning NOT: Client Needs Category: Health Promotion and Maintenance

7. A client has intra-arterial blood pressure monitoring after a myocardial infarction. The nurse notes the client's heart rate has increased from 88 to 110 beats/min, and the blood pressure dropped from 120/82 to 100/60 mm Hg. What action by the nurse is most appropriate? a. Allow the client to rest quietly. b. Assess the client for bleeding. c. Document the findings in the chart. d. Medicate the client for pain.

ANS: B A major complication related to intra-arterial blood pressure monitoring is hemorrhage from the insertion site. Since these vital signs are out of the normal range, are a change, and are consistent with blood loss, the nurse should assess the client for any bleeding associated with the arterial line. The nurse should document the findings after a full assessment. The client may or may not need pain medication and rest; the nurse first needs to rule out any emergent bleeding. DIF: Applying/Application REF: 771 KEY: Coronary artery disease| intra-arterial blood pressure monitoring| equipment safety| vitalsigns MSC: IntegratedProcess:NursingProcess:Assessment NOT: Client Needs Category: Physiological Integrity: Reduction of Risk Potential

7. After administering newly prescribed captopril (Capoten) to a client with heart failure, the nurse implements interventions to decrease complications. Which priority intervention should the nurse implement for this client? a. Provide food to decrease nausea and aid in absorption. b. Instruct the client to ask for assistance when rising from bed. c. Collaborate with unlicensed assistive personnel to bathe the client. d. Monitor potassium levels and check for symptoms of hypokalemia.

ANS: B Administration of the first dose of angiotensin-converting enzyme (ACE) inhibitors is often associated with hypotension, usually termed first-dose effect. The nurse should instruct the client to seek assistance before arising from bed to prevent injury from postural hypotension. ACE inhibitors do not need to be taken with food. Collaboration with unlicensed assistive personnel to provide hygiene is not a priority. The client should be encouraged to complete activities of daily living as independently as possible. The nurse should monitor for hyperkalemia, not hypokalemia, especially if the client has renal insufficiency secondary to heart failure. DIF: Applying/Application REF: 685 KEY: Heart failure| angiotensin-converting enzyme (ACE) inhibitor| medication| patient education MSC: Integrated Process: Nursing Process: Implementation NOT: Client Needs Category: Physiological Integrity: Pharmacological and Parenteral Therapies

2. A client received tissue plasminogen activator (t-PA) after a myocardial infarction and now is on an intravenous infusion of heparin. The client's spouse asks why the client needs this medication. What response by the nurse is best? a. "The t-PA didn't dissolve the entire coronary clot." b. "The heparin keeps that artery from getting blocked again." c. "Heparin keeps the blood as thin as possible for a longer time." d. "The heparin prevents a stroke from occurring as the t-PA wears off."

ANS: B After the original intracoronary clot has dissolved, large amounts of thrombin are released into the bloodstream, increasing the chance of the vessel reoccluding. The other statements are not accurate. Heparin is not a "blood thinner," although laypeople may refer to it as such. DIF: Understanding/Comprehension REF: 768 KEY: Coronary artery disease| thrombolytic agents| patient education MSC: IntegratedProcess:Teaching/Learning NOT: Client Needs Category: Physiological Integrity: Pharmacological and Parenteral Therapies

24. A nurse assesses a client who has mitral valve regurgitation. For which cardiac dysrhythmia should the nurse assess? a. Preventricular contractions b. Atrial fibrillation c. Symptomatic bradycardia d. Sinus tachycardia

ANS: B Atrial fibrillation is a clinical manifestation of mitral valve regurgitation and stenosis. Preventricular contractions and bradycardia are not associated with valvular problems. These are usually identified in clients with electrolyte imbalances, myocardial infarction, and sinus node problems. Sinus tachycardia is a manifestation of aortic regurgitation due to a decrease in cardiac output. DIF: Understanding/Comprehension REF: 692 KEY: Valve disorder| cardiac dysrhythmia MSC: IntegratedProcess:NursingProcess:Assessment NOT: Client Needs Category: Physiological Integrity: Reduction of Risk Potential

5. A nurse evaluates prescriptions for a client with chronic atrial fibrillation. Which medication should the nurse expect to find on this client's medication administration record to prevent a common complication of this condition? a. Sotalol (Betapace) b. Warfarin (Coumadin) c. Atropine (Sal-Tropine) d. Lidocaine (Xylocaine)

ANS: B Atrial fibrillation puts clients at risk for developing emboli. Clients at risk for emboli are treated with anticoagulants, such as heparin, enoxaparin, or warfarin. Sotalol, atropine, and lidocaine are not appropriate for this complication. DIF: Applying/Application REF: 667 KEY: Cardiac electrical conduction| medication MSC: Integrated Process: Nursing Process: Analysis NOT: Client Needs Category: Physiological Integrity: Pharmacological and Parenteral Therapies

2. A nurse cares for a client who has a heart rate averaging 56 beats/min with no adverse symptoms. Which activity modification should the nurse suggest to avoid further slowing of the heart rate? a. "Make certain that your bath water is warm." b. "Avoid straining while having a bowel movement." c. "Limit your intake of caffeinated drinks to one a day." d. "Avoid strenuous exercise such as running."

ANS: B Bearing down strenuously during a bowel movement is one type of Valsalva maneuver, which stimulates the vagus nerve and results in slowing of the heart rate. Such a response is not desirable in a person who has bradycardia. The other instructions are not appropriate for this condition. DIF: Applying/Application REF: 663 KEY: Functional ability MSC: Integrated Process: Nursing Process: Implementation NOT: Client Needs Category: Physiological Integrity: Basic Care and Comfort

14. After teaching a client who is being discharged home after mitral valve replacement surgery, the nurse assesses the client's understanding. Which client statement indicates a need for additional teaching? a. "I'll be able to carry heavy loads after 6 months of rest." b. "I will have my teeth cleaned by my dentist in 2 weeks." c. "I must avoid eating foods high in vitamin K, like spinach." d. "I must use an electric razor instead of a straight razor to shave."

ANS: B Clients who have defective or repaired valves are at high risk for endocarditis. The client who has had valve surgery should avoid dental procedures for 6 months because of the risk for endocarditis. When undergoing a mitral valve replacement surgery, the client needs to be placed on anticoagulant therapy to prevent vegetation forming on the new valve. Clients on anticoagulant therapy should be instructed on bleeding precautions, including using an electric razor. If the client is prescribed warfarin, the client should avoid foods high in vitamin K. Clients recovering from open heart valve replacements should not carry anything heavy for 6 months while the chest incision and muscle heal. DIF: Applying/Application REF: 694 KEY: Valve disorder| patient education| hygiene MSC: IntegratedProcess:Teaching/Learning NOT: Client Needs Category: Physiological Integrity: Reduction of Risk Potential

20. A nurse assesses a client who has a history of heart failure. Which question should the nurse ask to assess the extent of the client's heart failure? a. "Do you have trouble breathing or chest pain?" b. "Are you able to walk upstairs without fatigue?" c. "Do you awake with breathlessness during the night?" d. "Do you have new-onset heaviness in your legs?"

ANS: B Clients with a history of heart failure generally have negative findings, such as shortness of breath. The nurse needs to determine whether the client's activity is the same or worse, or whether the client identifies a decrease in activity level. Trouble breathing, chest pain, breathlessness at night, and peripheral edema are symptoms of heart failure, but do not provide data that can determine the extent of the client's heart failure. DIF: Applying/Application REF: 682 KEY: Heart failure| functional ability| respiratory distress/failure MSC: IntegratedProcess:NursingProcess:Assessment NOT: Client Needs Category: Physiological Integrity: Physiological Adaptation

4. A nurse assesses a client with atrial fibrillation. Which manifestation should alert the nurse to the possibility of a serious complication from this condition? a. Sinus tachycardia b. Speech alterations c. Fatigue d. Dyspnea with activity

ANS: B Clients with atrial fibrillation are at risk for embolic stroke. Evidence of embolic events includes changes in mentation, speech, sensory function, and motor function. Clients with atrial fibrillation often have a rapid ventricular response as a result. Fatigue is a nonspecific complaint. Clients with atrial fibrillation often have dyspnea as a result of the decreased cardiac output caused by the rhythm disturbance. DIF: Applying/Application REF: 666 KEY: Cardiac electrical conduction| vascular perfusion MSC: IntegratedProcess:NursingProcess:Assessment NOT: Client Needs Category: Physiological Integrity: Reduction of Risk Potential

12. A nurse assesses a client with mitral valve stenosis. What clinical manifestation should alert the nurse to the possibility that the client's stenosis has progressed? a. Oxygen saturation of 92% b. Dyspnea on exertion c. Muted systolic murmur d. Upper extremity weakness

ANS: B Dyspnea on exertion develops as the mitral valvular orifice narrows and pressure in the lungs increases. The other manifestations do not relate to the progression of mitral valve stenosis. DIF: Applying/Application REF: 688 KEY: Valve disorder| respiratory distress/failure MSC: IntegratedProcess:NursingProcess:Assessment NOT: Client Needs Category: Physiological Integrity: Reduction of Risk Potential

20. A home health care nurse is visiting an older client who lives alone after being discharged from the hospital after a coronary artery bypass graft. What finding in the home most causes the nurse to consider additional referrals? a. Dirty carpets in need of vacuuming b. Expired food in the refrigerator c. Old medications in the kitchen d. Several cats present in the home

ANS: B Expired food in the refrigerator demonstrates a safety concern for the client and a possible lack of money to buy food. The nurse can consider a referral to Meals on Wheels or another home-based food program. Dirty carpets may indicate the client has no household help and is waiting for clearance to vacuum. Old medications can be managed by the home health care nurse and the client working collaboratively. Having pets is not a cause for concern. DIF: Applying/Application REF: 781 KEY: Home safety| referrals| coronary artery bypass graft MSC: Integrated Process: Communication and Documentation NOT: Client Needs Category: Safe and Effective Care Environment: Safety and Infection Control

12. A nurse assists with the cardioversion of a client experiencing acute atrial fibrillation. Which action should the nurse take prior to the initiation of cardioversion? a. Administer intravenous adenosine. b. Turn off oxygen therapy. c. Ensure a tongue blade is available. d. Position the client on the left side.

ANS: B For safety during cardioversion, the nurse should turn off any oxygen therapy to prevent fire. The other interventions are not appropriate for a cardioversion. The client should be placed in a supine position. DIF: Remembering/Knowledge REF: 668 KEY: Assessment/diagnostic examination| safety MSC: Integrated Process: Nursing Process: Implementation NOT: Client Needs Category: Safe and Effective Care Environment: Safety and Infection Control

14. A nurse is in charge of the coronary intensive care unit. Which client should the nurse see first? a. Client on a nitroglycerin infusion at 5 mcg/min, not titrated in the last 4 hours b. Client who is 1 day post coronary artery bypass graft, blood pressure 180/100 mm Hg c. Client who is 1 day post percutaneous coronary intervention, going home this morning d. Client who is 2 days post coronary artery bypass graft, became dizzy this a.m. while walking

ANS: B Hypertension after coronary artery bypass graft surgery can be dangerous because it puts too much pressure on the suture lines and can cause bleeding. The charge nurse should see this client first. The client who became dizzy earlier should be seen next. The client on the nitroglycerin drip is stable. The client going home can wait until the other clients are cared for. DIF: Analyzing/Analysis REF: 777 KEY: Coronary artery disease| coronary artery bypass graft| collaboration MSC: IntegratedProcess:NursingProcess:Assessment NOT: Client Needs Category: Safe and Effective Care Environment: Management of Care

22. After assessing a client who is receiving an amiodarone intravenous infusion for unstable ventricular tachycardia, the nurse documents the findings and compares these with the previous assessment findings: Vital Signs Nursing Assessment Time: 0800 Temperature: 98° F Heart rate: 68 beats/min Blood pressure: 135/60 mm Hg Respiratory rate: 14 breaths/min Oxygen saturation: 96% Oxygen therapy: 2 L nasal cannula Time: 1000 Temperature: 98.2° F Heart rate: 50 beats/min Blood pressure: 132/57 mm Hg Respiratory rate: 16 breaths/min Oxygen saturation: 95% Oxygen therapy: 2 L nasal cannula Time: 0800 Client alert and oriented. Cardiac rhythm: normal sinus rhythm. Skin: warm, dry, and appropriate for race. Respirations equal and unlabored. Client denies shortness of breath and chest pain. Time: 1000 Client alert and oriented. Cardiac rhythm: sinus bradycardia. Skin: warm, dry, and appropriate for race. Respirations equal and unlabored. Client denies shortness of breath and chest pain. Client voids 420 mL of clear yellow urine. Based on the assessments, which action should the nurse take? a. Stop the infusion and flush the IV. b. Slow the amiodarone infusion rate. c. Administer IV normal saline. d. Ask the client to cough and deep breathe.

ANS: B IV administration of amiodarone may cause bradycardia and atrioventricular (AV) block. The correct action for the nurse to take at this time is to slow the infusion, because the client is asymptomatic and no evidence reveals AV block that might require pacing. Abruptly ceasing the medication could allow fatal dysrhythmias to occur. The administration of IV fluids and encouragement of coughing and deep breathing exercises are not indicated, and will not increase the client's heart rate. DIF: Applying/Application REF: 660 KEY: Cardiac electrical conduction| medication MSC: Integrated Process: Nursing Process: Implementation NOT: Client Needs Category: Physiological Integrity: Pharmacological and Parenteral Therapies

6. A client has hemodynamic monitoring after a myocardial infarction. What safety precaution does the nurse implement for this client? a. Document pulmonary artery wedge pressure (PAWP) readings and assess their trends. b. Ensure the balloon does not remain wedged. c. Keep the client on strict NPO status. d. Maintain the client in a semi-Fowler's position.

ANS: B If the balloon remains inflated, it can cause pulmonary infarction or rupture. The nurse should ensure the balloon remains deflated between PAWP readings. Documenting PAWP readings and assessing trends is an important nursing action related to hemodynamic monitoring, but is not specifically related to safety. The client does not have to be NPO while undergoing hemodynamic monitoring. Positioning may or may not affect readings. DIF: Applying/Application REF: 770 KEY: Coronary artery disease| hemodynamic monitoring| equipment safety MSC: Integrated Process: Nursing Process: Implementation NOT: Client Needs Category: Safe and Effective Care Environment: Safety and Infection Control

19. The nurse is caring for a client with a chest tube after a coronary artery bypass graft. The drainage slows significantly. What action by the nurse is most important? a. Increase the setting on the suction. b. Notify the provider immediately. c. Re-position the chest tube. d. Take the tubing apart to assess for clots.

ANS: B If the drainage in the chest tube decreases significantly and dramatically, the tube may be blocked by a clot. This could lead to cardiac tamponade. The nurse should notify the provider immediately. The nurse should not independently increase the suction, re-position the chest tube, or take the tubing apart. DIF: Applying/Application REF: 778 KEY: Coronary artery bypass graft| critical rescue| chest tubes| cardiovascular system MSC: Integrated Process: Communication and Documentation NOT: Client Needs Category: Safe and Effective Care Environment: Management of Care

8. A nurse cares for a client with an intravenous temporary pacemaker for bradycardia. The nurse observes the presence of a pacing spike but no QRS complex on the client's electrocardiogram. Which action should the nurse take next? a. Administer intravenous diltiazem (Cardizem). b. Assess vital signs and level of consciousness. c. Administer sublingual nitroglycerin. d. Assess capillary refill and temperature.

ANS: B In temporary pacing, the wires are threaded onto the epicardial surface of the heart and exit through the chest wall. The pacemaker spike should be followed immediately by a QRS complex. Pacing spikes seen without subsequent QRS complexes imply loss of capture. If there is no capture, then there is no ventricular depolarization and contraction. The nurse should assess for cardiac output via vital signs and level of consciousness. The other interventions would not determine if the client is tolerating the loss of capture. DIF: Applying/Application REF: 664 KEY: Cardiac electrical conduction MSC: IntegratedProcess:NursingProcess:Assessment NOT: Client Needs Category: Physiological Integrity: Reduction of Risk Potential

15. A client with coronary artery disease (CAD) asks the nurse about taking fish oil supplements. What response by the nurse is best? a. "Fish oil is contraindicated with most drugs for CAD." b. "The best source is fish, but pills have benefits too." c. "There is no evidence to support fish oil use with CAD." d. "You can reverse CAD totally with diet and supplements."

ANS: B Omega-3 fatty acids have shown benefit in reducing lipid levels, in reducing the incidence of sudden cardiac death, and for stabilizing atherosclerotic plaque. The best source is fish three times a week or some fish oil supplements. The other options are not accurate. DIF: Understanding/Comprehension REF: 761 KEY: Coronary artery disease| lipid-reducing agents| supplements| patient education MSC: IntegratedProcess:Teaching/Learning NOT: Client Needs Category: Health Promotion and Maintenance

3. A nurse assesses a client admitted to the cardiac unit. Which statement by the client alerts the nurse to the possibility of right-sided heart failure? a. "I sleep with four pillows at night." b. "My shoes fit really tight lately." c. "I wake up coughing every night." d. "I have trouble catching my breath."

ANS: B Signs of systemic congestion occur with right-sided heart failure. Fluid is retained, pressure builds in the venous system, and peripheral edema develops. Left-sided heart failure symptoms include respiratory symptoms. Orthopnea, coughing, and difficulty breathing all could be results of left-sided heart failure. DIF: Understanding/Comprehension REF: 683 KEY: Heart failure| assessment/diagnostic examination MSC: IntegratedProcess:NursingProcess:Assessment NOT: Client Needs Category: Health Promotion and Maintenance

13. A nurse cares for a client recovering from prosthetic valve replacement surgery. The client asks, "Why will I need to take anticoagulants for the rest of my life?" How should the nurse respond? a. "The prosthetic valve places you at greater risk for a heart attack." b. "Blood clots form more easily in artificial replacement valves." c. "The vein taken from your leg reduces circulation in the leg." d. "The surgery left a lot of small clots in your heart and lungs."

ANS: B Synthetic valve prostheses and scar tissue provide surfaces on which platelets can aggregate easily and initiate the formation of blood clots. The other responses are inaccurate. DIF: Applying/Application REF: 696 KEY: Valve disorder| patient education| anticoagulants MSC: IntegratedProcess:Teaching/Learning NOT: Client Needs Category: Physiological Integrity: Reduction of Risk Potential

16. A client has presented to the emergency department with an acute myocardial infarction (MI). What action by the nurse is best to meet The Joint Commission's Core Measures outcomes? a. Obtain an electrocardiogram (ECG) now and in the morning. b. Give the client an aspirin. c. Notify the Rapid Response Team. d. Prepare to administer thrombolytics.

ANS: B The Joint Commission's Core Measures set for acute MI require that aspirin is administered when a client with MI presents to the emergency department or when an MI occurs in the hospital. A rapid ECG is vital, but getting another one in the morning is not part of the Core Measures set. The Rapid Response Team is not needed if an emergency department provider is available. Thrombolytics may or may not be needed. DIF: Remembering/Knowledge REF: 766 KEY: Coronary artery disease| Core Measures| The Joint Commission MSC: Integrated Process: Nursing Process: Implementation NOT: Client Needs Category: Safe and Effective Care Environment: Management of Care

10. After teaching a client who has an implantable cardioverter-defibrillator (ICD), a nurse assesses the client's understanding. Which statement by the client indicates a correct understanding of the teaching? a. "I should wear a snug-fitting shirt over the ICD." b. "I will avoid sources of strong electromagnetic fields." c. "I should participate in a strenuous exercise program." d. "Now I can discontinue my antidysrhythmic medication."

ANS: B The client being discharged with an ICD is instructed to avoid strong sources of electromagnetic fields. Clients should avoid tight clothing, which could cause irritation over the ICD generator. The client should be encouraged to exercise but should not engage in strenuous activities that cause the heart rate to meet or exceed the ICD cutoff point because the ICD can discharge inappropriately. The client should continue all prescribed medications. DIF: Applying/Application REF: 674 KEY: Cardiac electrical conduction MSC: Integrated Process: Teaching/Learning NOT: Client Needs Category: Health Promotion and Maintenance

16. A nurse assesses a client with pericarditis. Which assessment finding should the nurse expect to find? a. Heart rate that speeds up and slows down b. Friction rub at the left lower sternal border c. Presence of a regular gallop rhythm d. Coarse crackles in bilateral lung bases

ANS: B The client with pericarditis may present with a pericardial friction rub at the left lower sternal border. This sound is the result of friction from inflamed pericardial layers when they rub together. The other assessments are not related. DIF: Remembering/Knowledge REF: 699 KEY: Inflammatory response| assessment/diagnostic examination MSC: IntegratedProcess:NursingProcess:Assessment NOT: Client Needs Category: Physiological Integrity: Physiological Adaptation

17. A nurse is caring for four clients. Which client should the nurse assess first? a. Client with an acute myocardial infarction, pulse 102 beats/min b. Client who is 1 hour post angioplasty, has tongue swelling and anxiety c. Client who is post coronary artery bypass, chest tube drained 100 mL/hr d. Client who is post coronary artery bypass, potassium 4.2 mEq/L

ANS: B The post-angioplasty client with tongue swelling and anxiety is exhibiting manifestations of an allergic reaction that could progress to anaphylaxis. The nurse should assess this client first. The client with a heart rate of 102 beats/min may have increased oxygen demands but is just over the normal limit for heart rate. The two post coronary artery bypass clients are stable. DIF: Analyzing/Analysis REF: 774 KEY: Coronary artery disease| critical rescue| medical emergencies| hypersensitivities| allergic reaction MSC: Integrated Process: Nursing Process: Analysis NOT: Client Needs Category: Safe and Effective Care Environment: Management of Care

18. A nurse teaches a client recovering from a heart transplant who is prescribed cyclosporine (Sandimmune). Which statement should the nurse include in this client's discharge teaching? a. "Use a soft-bristled toothbrush and avoid flossing." b. "Avoid large crowds and people who are sick." c. "Change positions slowly to avoid hypotension." d. "Check your heart rate before taking the medication."

ANS: B These agents cause immune suppression, leaving the client more vulnerable to infection. The medication does not place the client at risk for bleeding, orthostatic hypotension, or a change in heart rate. DIF: Applying/Application REF: 703 KEY: Transplant| immune suppressant MSC: Integrated Process: Nursing Process: Implementation NOT: Client Needs Category: Physiological Integrity: Pharmacological and Parenteral Therapies

3. A client is in the hospital after suffering a myocardial infarction and has bathroom privileges. The nurse assists the client to the bathroom and notes the client's O2 saturation to be 95%, pulse 88 beats/min, and respiratory rate 16 breaths/min after returning to bed. What action by the nurse is best? a. Administer oxygen at 2 L/min. b. Allow continued bathroom privileges. c. Obtain a bedside commode. d. Suggest the client use a bedpan.

ANS: B This client's physiologic parameters did not exceed normal during and after activity, so it is safe for the client to continue using the bathroom. There is no indication that the client needs oxygen, a commode, or a bedpan. DIF: Applying/Application REF: 769 KEY: Coronary artery disease| activity intolerance| vital signs| nursing assessment MSC: IntegratedProcess:NursingProcess:Assessment NOT: Client Needs Category: Physiological Integrity: Reduction of Risk Potential

MULTIPLE RESPONSE 1. A nursing student learns about modifiable risk factors for coronary artery disease. Which factors does this include? (Select all that apply.) a. Age b. Hypertension c. Obesity d. Smoking e. Stress

ANS: B, C, D, E Hypertension, obesity, smoking, and excessive stress are all modifiable risk factors for coronary artery disease. Age is a nonmodifiable risk factor. DIF: Remembering/Knowledge REF: 760 KEY: Coronary artery disease| lifestyle factors MSC: Integrated Process: Teaching/Learning NOT: Client Needs Category: Health Promotion and Maintenance

9. A nurse assesses a client who is diagnosed with infective endocarditis. Which assessment findings should the nurse expect? (Select all that apply.) a. Weight gain b. Night sweats c. Cardiac murmur d. Abdominal bloating e. Osler's nodes

ANS: B, C, E Clinical manifestations of infective endocarditis include fever with chills, night sweats, malaise and fatigue, anorexia and weight loss, cardiac murmur, and Osler's nodes on palms of the hands and soles of the feet. Abdominal bloating is a manifestation of heart transplantation rejection. DIF: Remembering/Knowledge REF: 697 KEY: Endocarditis MSC: IntegratedProcess:NursingProcess:Assessment NOT: Client Needs Category: Physiological Integrity: Physiological Adaptation

4. A client is 1 day postoperative after a coronary artery bypass graft. What nonpharmacologic comfort measures does the nurse include when caring for this client? (Select all that apply.) a. Administer pain medication before ambulating. b. Assist the client into a position of comfort in bed. c. Encourage high-protein diet selections. d. Provide complementary therapies such as music. e. Remind the client to splint the incision when coughing.

ANS: B, D, E Nonpharmacologic comfort measures can include positioning, complementary therapies, and splinting the chest incision. Medications are not nonpharmacologic. Food choices are not comfort measures. DIF: Applying/Application REF: 781 KEY: Coronary artery disease| nonpharmacologic comfort measures MSC: Integrated Process: Nursing Process: Implementation NOT: Client Needs Category: Physiological Integrity: Basic Care and Comfort

1. A client is receiving an infusion of tissue plasminogen activator (t-PA). The nurse assesses the client to be disoriented to person, place, and time. What action by the nurse is best? a. Assess the client's pupillary responses. b. Request a neurologic consultation. c. Stop the infusion and call the provider. d. Take and document a full set of vital signs.

ANS: C A change in neurologic status in a client receiving t-PA could indicate intracranial hemorrhage. The nurse should stop the infusion and notify the provider immediately. A full assessment, including pupillary responses and vital signs, occurs next. The nurse may or may not need to call a neurologist. DIF: Applying/Application REF: 768 KEY: Coronary artery disease| neurologic system| critical rescue| Rapid Response Team| thrombolytic agents MSC: Integrated Process: Nursing Process: Implementation NOT: Client Needs Category: Physiological Integrity: Pharmacological and Parenteral Therapies

7. A telemetry nurse assesses a client with third-degree heart block who has wide QRS complexes and a heart rate of 35 beats/min on the cardiac monitor. Which assessment should the nurse complete next? a. Pulmonary auscultation b. Pulse strength and amplitude c. Level of consciousness d. Mobility and gait stability

ANS: C A heart rate of 40 beats/min or less with widened QRS complexes could have hemodynamic consequences. The client is at risk for inadequate cerebral perfusion. The nurse should assess for level of consciousness, light-headedness, confusion, syncope, and seizure activity. Although the other assessments should be completed, the client's level of consciousness is the priority. DIF: Applying/Application REF: 670 KEY: Cardiac electrical conduction| vascular perfusion MSC: IntegratedProcess:NursingProcess:Assessment NOT: Client Needs Category: Physiological Integrity: Reduction of Risk Potential

24. The provider requests the nurse start an infusion of an inotropic agent on a client. How does the nurse explain the action of these drugs to the client and spouse? a. "It constricts vessels, improving blood flow." b. "It dilates vessels, which lessens the work of the heart." c. "It increases the force of the heart's contractions." d. "It slows the heart rate down for better filling."

ANS: C A positive inotrope is a medication that increases the strength of the heart's contractions. The other options are not correct. DIF: Remembering/Knowledge REF: 772 KEY: Coronary artery disease| inotropic agents| patient education MSC: IntegratedProcess:Teaching/Learning NOT: Client Needs Category: Physiological Integrity: Pharmacological and Parenteral Therapies

10. A client in the cardiac stepdown unit reports severe, crushing chest pain accompanied by nausea and vomiting. What action by the nurse takes priority? a. Administer an aspirin. b. Call for an electrocardiogram (ECG). c. Maintain airway patency. d. Notify the provider.

ANS: C Airway always is the priority. The other actions are important in this situation as well, but the nurse should stay with the client and ensure the airway remains patent (especially if vomiting occurs) while another person calls the provider (or Rapid Response Team) and facilitates getting an ECG done. Aspirin will probably be administered, depending on the provider's prescription and the client's current medications. DIF: Applying/Application REF: 769 KEY: Coronary artery disease| critical rescue| medical emergencies MSC: Integrated Process: Nursing Process: Implementation NOT: Client Needs Category: Safe and Effective Care Environment: Management of Care

17. After teaching a client who is recovering from a heart transplant to change positions slowly, the client asks, "Why is this important?" How should the nurse respond? a. "Rapid position changes can create shear and friction forces, which can tear out your internal vascular sutures." b. "Your new vascular connections are more sensitive to position changes, leading to increased intravascular pressure and dizziness." c. "Your new heart is not connected to the nervous system and is unable to respond to decreases in blood pressure caused by position changes." d. "While your heart is recovering, blood flow is diverted away from the brain, increasing the risk for stroke when you stand up."

ANS: C Because the new heart is denervated, the baroreceptor and other mechanisms that compensate for blood pressure drops caused by position changes do not function. This allows orthostatic hypotension to persist in the postoperative period. The other options are false statements and do not correctly address the client's question. DIF: Understanding/Comprehension REF: 703 KEY: Transplant| patient education MSC: IntegratedProcess:NursingProcess:Implementation NOT: Client Needs Category: Physiological Integrity: Physiological Adaptation

6. A nurse administers prescribed adenosine (Adenocard) to a client. Which response should the nurse assess for as the expected therapeutic response? a. Decreased intraocular pressure b. Increased heart rate c. Short period of asystole d. Hypertensive crisis

ANS: C Clients usually respond to adenosine with a short period of asystole, bradycardia, hypotension, dyspnea, and chest pain. Adenosine has no conclusive impact on intraocular pressure. DIF: Applying/Application REF: 662 KEY: Cardiac electrical conduction| medication MSC: Integrated Process: Nursing Process: Analysis NOT: Client Needs Category: Physiological Integrity: Pharmacological and Parenteral Therapies

11. A nurse cares for a client with atrial fibrillation who reports fatigue when completing activities of daily living. What interventions should the nurse implement to address this client's concerns? a. Administer oxygen therapy at 2 liters per nasal cannula. b. Provide the client with a sleeping pill to stimulate rest. c. Schedule periods of exercise and rest during the day. d. Ask unlicensed assistive personnel to help bathe the client.

ANS: C Clients who have atrial fibrillation are at risk for decreased cardiac output and fatigue when completing activities of daily living. The nurse should schedule periods of exercise and rest during the day to decrease fatigue. The other interventions will not assist the client with self-care activities. DIF: Applying/Application REF: 658 KEY: Cardiac electrical conduction MSC: Integrated Process: Teaching/Learning NOT: Client Needs Category: Physiological Integrity: Basic Care and Comfort

16. The nurse asks a client who has experienced ventricular dysrhythmias about substance abuse. The client asks, "Why do you want to know if I use cocaine?" How should the nurse respond? a. "Substance abuse puts clients at risk for many health issues." b. "The hospital requires that I ask you about cocaine use." c. "Clients who use cocaine are at risk for fatal dysrhythmias." d. "We can provide services for cessation of substance abuse."

ANS: C Clients who use cocaine or illicit inhalants are particularly at risk for potentially fatal dysrhythmias. The other responses do not adequately address the client's question. DIF: Remembering/Knowledge REF: 657 KEY: Cardiac electrical conduction| substance abuse MSC: IntegratedProcess:NursingProcess:Assessment NOT: Client Needs Category: Psychosocial Integrity

2. A nurse assesses a client in an outpatient clinic. Which statement alerts the nurse to the possibility of left-sided heart failure? a. "I have been drinking more water than usual." b. "I am awakened by the need to urinate at night." c. "I must stop halfway up the stairs to catch my breath." d. "I have experienced blurred vision on several occasions."

ANS: C Clients with left-sided heart failure report weakness or fatigue while performing normal activities of daily living, as well as difficulty breathing, or "catching their breath." This occurs as fluid moves into the alveoli. Nocturia is often seen with right-sided heart failure. Thirst and blurred vision are not related to heart failure. DIF: Understanding/Comprehension REF: 682 KEY: Heart failure| assessment/diagnostic examination MSC: IntegratedProcess:NursingProcess:Assessment NOT: Client Needs Category: Health Promotion and Maintenance

13. A client has an intra-arterial blood pressure monitoring line. The nurse notes bright red blood on the client's sheets. What action should the nurse perform first? a. Assess the insertion site. b. Change the client's sheets. c. Put on a pair of gloves. d. Assess blood pressure.

ANS: C For the nurse's safety, he or she should put on a pair of gloves to prevent blood exposure. The other actions are appropriate as well, but first the nurse must don a pair of gloves. DIF: Applying/Application REF: 771 KEY: Standard Precautions| infection control| intra-arterial blood pressure monitoring| staff safety MSC: Integrated Process: Nursing Process: Implementation NOT: Client Needs Category: Safe and Effective Care Environment: Safety and Infection Control

22. A client had an acute myocardial infarction. What assessment finding indicates to the nurse that a significant complication has occurred? a. Blood pressure that is 20 mm Hg below baseline b. Oxygen saturation of 94% on room air c. Poor peripheral pulses and cool skin d. Urine output of 1.2 mL/kg/hr for 4 hours

ANS: C Poor peripheral pulses and cool skin may be signs of impending cardiogenic shock and should be reported immediately. A blood pressure drop of 20 mm Hg is not worrisome. An oxygen saturation of 94% is just slightly below normal. A urine output of 1.2 mL/kg/hr for 4 hours is normal. DIF: Remembering/Knowledge REF: 772 KEY: Coronary artery disease| critical rescue| nursing assessment MSC: Integrated Process: Nursing Process: Analysis NOT: Client Needs Category: Physiological Integrity: Reduction of Risk Potential

23. A client presents to the emergency department with an acute myocardial infarction (MI) at 1500 (3:00 PM). The facility has 24-hour catheterization laboratory abilities. To meet The Joint Commission's Core Measures set, by what time should the client have a percutaneous coronary intervention performed? a. 1530 (3:30 PM) b. 1600 (4:00 PM) c. 1630 (4:30 PM) d. 1700 (5:00 PM)

ANS: C The Joint Commission's Core Measures set for MI includes percutaneous coronary intervention within 90 minutes of diagnosis of myocardial infarction. Therefore, the client should have a percutaneous coronary intervention performed no later than 1630 (4:30 PM). DIF: Remembering/Knowledge REF: 774 KEY: Coronary artery disease| Core Measures| The Joint Commission MSC: Integrated Process: Communication and Documentation NOT: Client Needs Category: Safe and Effective Care Environment: Management of Care

20. A nurse performs an admission assessment on a 75-year-old client with multiple chronic diseases. The client's blood pressure is 135/75 mm Hg and oxygen saturation is 94% on 2 liters per nasal cannula. The nurse assesses the client's rhythm on the cardiac monitor and observes the reading shown below: Which action should the nurse take first? a. Begin external temporary pacing. b. Assess peripheral pulse strength. c. Ask the client what medications he or she takes. d. Administer 1 mg of atropine.

ANS: C This client is stable and therefore does not require any intervention except to determine the cause of the bradycardia. Bradycardia is often caused by medications. Clients who have multiple chronic diseases are often on multiple medications that can interact with each other. The nurse should assess the client's current medications first. DIF: Applying/Application REF: 658 KEY: Cardiac electrical conduction| medications| adverse effects MSC: IntegratedProcess:NursingProcess:Assessment NOT: Client Needs Category: Physiological Integrity: Pharmacological and Parenteral Therapies

8. A nurse assesses a client after administering isosorbide mononitrate (Imdur). The client reports a headache. Which action should the nurse take? a. Initiate oxygen therapy. b. Hold the next dose of Imdur. c. Instruct the client to drink water. d. Administer PRN acetaminophen.

ANS: D The vasodilating effects of isosorbide mononitrate frequently cause clients to have headaches during the initial period of therapy. Clients should be told about this side effect and encouraged to take the medication with food. Some clients obtain relief with mild analgesics, such as acetaminophen. The client's headache is not related to hypoxia or dehydration; therefore, these interventions would not help. The client needs to take the medication as prescribed to prevent angina; the medication should not be held. DIF: Applying/Application REF: 686 KEY: Heart failure| nitroglycerin/nitrates| medication| pharmacologic pain management MSC: Integrated Process: Nursing Process: Implementation NOT: Client Needs Category: Physiological Integrity: Pharmacological and Parenteral Therapies

10. A nurse teaches a client who has a history of heart failure. Which statement should the nurse include in this client's discharge teaching? a. "Avoid drinking more than 3 quarts of liquids each day." b. "Eat six small meals daily instead of three larger meals." c. "When you feel short of breath, take an additional diuretic." d. "Weigh yourself daily while wearing the same amount of clothing."

ANS: D Clients with heart failure are instructed to weigh themselves daily to detect worsening heart failure early, and thus avoid complications. Other signs of worsening heart failure include increasing dyspnea, exercise intolerance, cold symptoms, and nocturia. Fluid overload increases symptoms of heart failure. The client should be taught to eat a heart-healthy diet, balance intake and output to prevent dehydration and overload, and take medications as prescribed. The most important discharge teaching is daily weights as this provides the best data related to fluid retention. DIF: Applying/Application REF: 687 KEY: Heart failure| patient education MSC: Integrated Process: Teaching/Learning NOT: Client Needs Category: Health Promotion and Maintenance

9. A nurse teaches a client who is prescribed digoxin (Lanoxin) therapy. Which statement should the nurse include in this client's teaching? a. "Avoid taking aspirin or aspirin-containing products." b. "Increase your intake of foods that are high in potassium." c. "Hold this medication if your pulse rate is below 80 beats/min." d. "Do not take this medication within 1 hour of taking an antacid."

ANS: D Gastrointestinal absorption of digoxin is erratic. Many medications, especially antacids, interfere with its absorption. Clients are taught to hold their digoxin for bradycardia; a heart rate of 80 beats/min is too high for this cutoff. Potassium and aspirin have no impact on digoxin absorption, nor do these statements decrease complications of digoxin therapy. DIF: Applying/Application REF: 686 KEY: Heart failure| digoxin| medication| patient education MSC: Integrated Process: Nursing Process: Implementation NOT: Client Needs Category: Physiological Integrity: Pharmacological and Parenteral Therapies

1. A nurse assesses a client's electrocardiograph tracing and observes that not all QRS complexes are preceded by a P wave. How should the nurse interpret this observation? a. The client has hyperkalemia causing irregular QRS complexes. b. Ventricular tachycardia is overriding the normal atrial rhythm. c. The client's chest leads are not making sufficient contact with the skin. d. Ventricular and atrial depolarizations are initiated from different sites.

ANS: D Normal rhythm shows one P wave preceding each QRS complex, indicating that all depolarization is initiated at the sinoatrial node. QRS complexes without a P wave indicate a different source of initiation of depolarization. This finding on an electrocardiograph tracing is not an indication of hyperkalemia, ventricular tachycardia, or disconnection of leads. DIF: Understanding/Comprehension REF: 649 KEY: Cardiac electrical conduction MSC: Integrated Process: Nursing Process: Analysis NOT: Client Needs Category: Physiological Integrity: Physiological Adaptation

5. A client undergoing hemodynamic monitoring after a myocardial infarction has a right atrial pressure of 0.5 mm Hg. What action by the nurse is most appropriate? a. Level the transducer at the phlebostatic axis. b. Lay the client in the supine position. c. Prepare to administer diuretics. d. Prepare to administer a fluid bolus.

ANS: D Normal right atrial pressures are from 1 to 8 mm Hg. Lower pressures usually indicate hypovolemia, so the nurse should prepare to administer a fluid bolus. The transducer should remain leveled at the phlebostatic axis. Positioning may or may not influence readings. Diuretics would be contraindicated. DIF: Applying/Application REF: 770 KEY: Coronary artery disease| hemodynamic monitoring| fluid and electrolyte imbalance MSC: Integrated Process: Nursing Process: Implementation NOT: Client Needs Category: Physiological Integrity: Physiological Adaptation

23. A nurse is caring for a client with acute pericarditis who reports substernal precordial pain that radiates to the left side of the neck. Which nonpharmacologic comfort measure should the nurse implement? a. Apply an ice pack to the client's chest. b. Provide a neck rub, especially on the left side. c. Allow the client to lie in bed with the lights down. d. Sit the client up with a pillow to lean forward on.

ANS: D Pain from acute pericarditis may worsen when the client lays supine. The nurse should position the client in a comfortable position, which usually is upright and leaning slightly forward. Pain is decreased by using gravity to take pressure off the heart muscle. An ice pack and neck rub will not relieve this pain. DIF: Applying/Application REF: 699 KEY: Nonpharmacologic pain management MSC: Integrated Process: Nursing Process: Implementation NOT: Client Needs Category: Physiological Integrity: Basic Care and Comfort

8. A client is in the preoperative holding area prior to an emergency coronary artery bypass graft (CABG). The client is yelling at family members and tells the doctor to "just get this over with" when asked to sign the consent form. What action by the nurse is best? a. Ask the family members to wait in the waiting area. b. Inform the client that this behavior is unacceptable. c. Stay out of the room to decrease the client's stress levels. d. Tell the client that anxiety is common and that you can help.

ANS: D Preoperative fear and anxiety are common prior to cardiac surgery, especially in emergent situations. The client is exhibiting anxiety, and the nurse should reassure the client that fear is common and offer to help. The other actions will not reduce the client's anxiety. DIF: Applying/Application REF: 776 KEY: Coronary artery disease| preoperative nursing| psychosocial response| anxiety| coping| therapeutic communication MSC: Integrated Process: Caring NOT: Client Needs Category: Psychosocial Integrity

18. A nurse assesses a client's electrocardiogram (ECG) and observes the reading shown below: How should the nurse document this client's ECG strip? a. Ventricular tachycardia b. V entricular fibrillation c. Sinus rhythm with premature atrial contractions (PACs) d. Sinus rhythm with premature ventricular contractions (PVCs)

ANS: D Sinus rhythm with PVCs has an underlying regular sinus rhythm with ventricular depolarization that sometimes precede atrial depolarization. Ventricular tachycardia and ventricular fibrillation rhythms would not have sinus beats present. Premature atrial contractions are atrial contractions initiated from another region of the atria before the sinus node initiates atrial depolarization. DIF: Applying/Application REF: 669 KEY: Cardiac electrical conduction| documentation MSC: Integrated Process: Communication and Documentation NOT: Client Needs Category: Physiological Integrity: Physiological Adaptation

19. A nurse cares for a client with end-stage heart failure who is awaiting a transplant. The client appears depressed and states, "I know a transplant is my last chance, but I don't want to become a vegetable." How should the nurse respond? a. "Would you like to speak with a priest or chaplain?" b. "I will arrange for a psychiatrist to speak with you." c. "Do you want to come off the transplant list?" d. "Would you like information about advance directives?"

ANS: D The client is verbalizing a real concern or fear about negative outcomes of the surgery. This anxiety itself can have a negative effect on the outcome of the surgery because of sympathetic stimulation. The best action is to allow the client to verbalize the concern and work toward a positive outcome without making the client feel as though he or she is crazy. The client needs to feel that he or she has some control over the future. The nurse personally provides care to address the client's concerns instead of pushing the client's issues off on a chaplain or psychiatrist. The nurse should not jump to conclusions and suggest taking the client off the transplant list, which is the best treatment option. DIF: Applying/Application REF: 691 KEY: Transplant| psychosocial response| anxiety MSC: Integrated Process: Nursing Process: Implementation NOT: Client Needs Category: Psychosocial Integrity

25. A nurse is assessing a client who had a myocardial infarction. Upon auscultating heart sounds, the nurse hears the following sound. What action by the nurse is most appropriate? (Click the media button to hear the audio clip.) a. Assess for further chest pain. b. Call the Rapid Response Team. c. Have the client sit upright. d. Listen to the client's lung sounds.

ANS: D The sound the nurse hears is an S3 heart sound, an abnormal sound that may indicate heart failure. The nurse should next assess the client's lung sounds. Assessing for chest pain is not directly related. There is no indication that the Rapid Response Team is needed. Having the client sit up will not change the heart sound. DIF: Applying/Application REF: 762 KEY: Coronary artery disease| respiratory assessment| respiratory system| nursing assessment MSC: IntegratedProcess:NursingProcess:Assessment NOT: Client Needs Category: Physiological Integrity: Reduction of Risk Potential

9. A nurse prepares to defibrillate a client who is in ventricular fibrillation. Which priority intervention should the nurse perform prior to defibrillating this client? a. Make sure the defibrillator is set to the synchronous mode. b. Administer 1 mg of intravenous epinephrine. c. Test the equipment by delivering a smaller shock at 100 joules. d. Ensure that everyone is clear of contact with the client and the bed.

ANS: D To avoid injury, the rescuer commands that all personnel clear contact with the client or the bed and ensures their compliance before delivery of the shock. A precordial thump can be delivered when no defibrillator is available. Defibrillation is done in asynchronous mode. Equipment should not be tested before a client is defibrillated because this is an emergency procedure; equipment should be checked on a routine basis. Epinephrine should be administered after defibrillation. DIF: Applying/Application REF: 668 KEY: Cardiac electrical conduction| safety MSC: Integrated Process: Nursing Process: Implementation NOT: Client Needs Category: Safe and Effective Care Environment: Safety and Infection Control

12. The nurse is preparing to change a client's sternal dressing. What action by the nurse is most important? a. Assess vital signs. b. Don a mask and gown. c. Gather needed supplies. d. Perform hand hygiene.

ANS: D To prevent a sternal wound infection, the nurse washes hands or performs hand hygiene as a priority. Vital signs do not necessarily need to be assessed beforehand. A mask and gown are not needed. The nurse should gather needed supplies, but this is not the priority. DIF: Applying/Application REF: 776 KEY: Coronary artery disease| infection control| hand hygiene MSC: Integrated Process: Nursing Process: Implementation NOT: Client Needs Category: Safe and Effective Care Environment: Safety and Infection Control

The nurse is caring for a group of clients who have sustained myocardial infarction (MI). The nurse observes the client with which type of MI most carefully for the development of left ventricular heart failure? Inferior wall Anterior wall Lateral wall Posterior wall

Anterior wall Due to the large size of the anterior wall, the amount of tissue infarction may be large enough to decrease the force of contraction, leading to heart failure. The client with an inferior wall MI is more likely to develop right ventricular heart failure. Clients with obstruction of the circumflex artery may experience a lateral wall MI and sinus dysrhythmias or a posterior wall MI and sinus dysrhythmias.

An older adult client, 4 hours after coronary artery bypass graft (CABG), has a blood pressure of 80/50 mm Hg. What action does the nurse take? No action is required; low blood pressure is normal for older adults. No action is required for postsurgical CABG clients. Assess pulmonary artery wedge pressure (PAWP). Give ordered loop diuretics.

Assess pulmonary artery wedge pressure (PAWP). Decreased preload as exhibited by decreased PAWP could indicate hypovolemia secondary to hemorrhage or vasodilation; hypotension could cause the graft to collapse. Low blood pressure is not normal in older adults or postoperative clients. The cause of hypotension must be found and treated; further action is needed to determine additional interventions. Hypotension could be caused by hypovolemia; giving loop diuretics increases hypovolemia.

14. A nurse teaches a client with diabetes mellitus and a body mass index of 42 who is at high risk for coronary artery disease. Which statement related to nutrition should the nurse include in this clients teaching? a. The best way to lose weight is a high-protein, low-carbohydrate diet. b. You should balance weight loss with consuming necessary nutrients. c. A nutritionist will provide you with information about your new diet. d. If you exercise more frequently, you wont need to change your diet.

B Clients at risk for cardiovascular diseases should follow the American Heart Association guidelines to combat obesity and improve cardiac health. The nurse should encourage the client to eat vegetables, fruits, unrefined whole-grain products, and fat-free dairy products while losing weight. High-protein food items are often high in fat and calories. Although the nutritionist can assist with client education, the nurse should include nutrition education and assist the client to make healthy decisions. Exercising and eating nutrient-rich foods are both important components in reducing cardiovascular risk.

13. A nurse cares for a client who has an 80% blockage of the right coronary artery (RCA) and is scheduled for bypass surgery. Which intervention should the nurse be prepared to implement while this client waits for surgery? a. Administration of IV furosemide (Lasix) b. Initiation of an external pacemaker c. Assistance with endotracheal intubation d. Placement of central venous access

B The RCA supplies the right atrium, the right ventricle, the inferior portion of the left ventricle, and the atrioventricular (AV) node. It also supplies the sinoatrial node in 50% of people. If the client totally occludes the RCA, the AV node would not function and the client would go into heart block, so emergency pacing should be available for the client. Furosemide, intubation, and central venous access will not address the primary complication of RCA occlusion, which is AV node malfunction.

11. A nurse cares for a client who is prescribed magnetic resonance imaging (MRI) of the heart. The clients health history includes a previous myocardial infarction and pacemaker implantation. Which action should the nurse take? a. Schedule an electrocardiogram just before the MRI. b. Notify the health care provider before scheduling the MRI. c. Call the physician and request a laboratory draw for cardiac enzymes. d. Instruct the client to increase fluid intake the day before the MRI.

B The magnetic fields of the MRI can deactivate the pacemaker. The nurse should call the health care provider and report that the client has a pacemaker so the provider can order other diagnostic tests. The client does not need an electrocardiogram, cardiac enzymes, or increased fluids.

20. A nurse auscultated heart tones on an older adult client. Which action should the nurse take based on heart tones heard? (Click the media button to hear the audio clip.) a. Administer a diuretic. b. Document the finding. c. Decrease the IV flow rate. d. Evaluate the clients medications.

B The sound heard is an atrial gallop S4. An atrial gallop may be heard in older clients because of a stiffened ventricle. The nurse should document the finding, but no other intervention is needed at this time.

6. A nurse obtains the health history of a client who is newly admitted to the medical unit. Which statement by the client should alert the nurse to the presence of edema? a. I wake up to go to the bathroom at night. b. My shoes fit tighter by the end of the day. c. I seem to be feeling more anxious lately. d. I drink at least eight glasses of water a day.

B Weight gain can result from fluid accumulation in the interstitial spaces. This is known as edema. The nurse should note whether the client feels that his or her shoes or rings are tight, and should observe, when present, an indentation around the leg where the socks end. The other answers do not describe edema.

2. An emergency room nurse assesses a female client. Which assessment findings should alert the nurse to request a prescription for an electrocardiogram? (Select all that apply.) a. Hypertension b. Fatigue despite adequate rest c. Indigestion d. Abdominal pain e. Shortness of breath

B, C, E Women may not have chest pain with myocardial infarction, but may feel discomfort or indigestion. They often present with a triad of symptomsindigestion or feeling of abdominal fullness, feeling of chronic fatigue despite adequate rest, and feeling unable to catch their breath. Frequently, women are not diagnosed and therefore are not treated adequately. Hypertension and abdominal pain are not associated with acute coronary syndrome.

5. A nurse prepares a client for a pharmacologic stress echocardiogram. Which actions should the nurse take when preparing this client for the procedure? (Select all that apply.) a. Assist the provider to place a central venous access device. b. Prepare for continuous blood pressure and pulse monitoring. c. Administer the clients prescribed beta blocker. d. Give the client nothing by mouth 3 to 6 hours before the procedure. e. Explain to the client that dobutamine will simulate exercise for this examination.

B, D, E Clients receiving a pharmacologic stress echocardiogram will need peripheral venous access and continuous blood pressure and pulse monitoring. The client must be NPO 3 to 6 hours prior to the procedure. Education about dobutamine, which will be administered during the procedure, should be performed. Beta blockers are often held prior to the procedure.

3. A nurse assesses a client who is recovering after a coronary catheterization. Which assessment findings in the first few hours after the procedure require immediate action by the nurse? (Select all that apply.) a. Blood pressure of 140/88 mm Hg b. Serum potassium of 2.9 mEq/L c. Warmth and redness at the site d. Expanding groin hematoma e. Rhythm changes on the cardiac monitor

B, D, E In the first few hours postprocedure, the nurse monitors for complications such as bleeding from the insertion site, hypotension, acute closure of the vessel, dye reaction, hypokalemia, and dysrhythmias. The clients blood pressure is slightly elevated but does not need immediate action. Warmth and redness at the site would indicate an infection, but this would not be present in the first few hours.

an older client has a history of coronary artery disease. which modifiable risk factors will the nurse assess to guide the client's teaching plan? select all that apply. A. older age B. tobacco use C. female D. high-fat diet E. family history F. obesity

B, D, F

the nurse assesses a client who had a coronary artery bypass graft yesterday. which assessment finding will the nurse report to the surgeon immediately? A. incisional pain B. BP of 136/76 C. decreased level of consciousness D. apical pulse of 88

C

9. A nurse assesses a client who is recovering after a left-sided cardiac catheterization. Which assessment finding requires immediate intervention? a. Urinary output less than intake b. Bruising at the insertion site c. Slurred speech and confusion d. Discomfort in the left leg

C A left-sided cardiac catheterization specifically increases the risk for a cerebral vascular accident. A change in neurologic status needs to be acted on immediately. Discomfort and bruising are expected at the site. If intake decreases, a client can become dehydrated because of dye excretion. The second intervention would be to increase the clients fluid status. Neurologic changes would take priority.

16. A nurse cares for a client who is recovering from a myocardial infarction. The client states, I will need to stop eating so much chili to keep that indigestion pain from returning. How should the nurse respond? a. Chili is high in fat and calories; it would be a good idea to stop eating it. b. The provider has prescribed an antacid for you to take every morning. c. What do you understand about what happened to you? d. When did you start experiencing this indigestion?

C Clients who experience myocardial infarction often respond with denial, which is a defense mechanism. The nurse should ask the client what he or she thinks happened, or what the illness means to him or her. The other responses do not address the clients misconception about recent pain and the cause of that pain.

7. A nurse assesses an older adult client who is experiencing a myocardial infarction. Which clinical manifestation should the nurse expect? a. Excruciating pain on inspiration b. Left lateral chest wall pain c. Disorientation and confusion d. Numbness and tingling of the arm

C In older adults, disorientation or confusion may be the major manifestation of myocardial infarction caused by poor cardiac output. Pain manifestations and numbness and tingling of the arm could also be related to the myocardial infarction. However, the nurse should be more concerned about the new onset of disorientation or confusion caused by decreased perfusion.

8. A nurse assesses a client 2 hours after a cardiac angiography via the left femoral artery. The nurse notes that the left pedal pulse is weak. Which action should the nurse take? a. Elevate the leg and apply a sandbag to the entrance site. b. Increase the flow rate of intravenous fluids. c. Assess the color and temperature of the left leg. d. Document the finding as left pedal pulse of +1/4.

C Loss of a pulse distal to an angiography entry site is serious, indicating a possible arterial obstruction. The pulse may be faint because of edema. The left pulse should be compared with the right, and pulses should be compared with previous assessments, especially before the procedure. Assessing color (pale, cyanosis) and temperature (cool, cold) will identify a decrease in circulation. Once all peripheral and vascular assessment data are acquired, the primary health care provider should be notified. Simply documenting the findings is inappropriate. The leg should be positioned below the level of the heart or dangling to increase blood flow to the distal portion of the leg. Increasing intravenous fluids will not address the clients problem.

4. A nurse assesses an older adult client who has multiple chronic diseases. The clients heart rate is 48 beats/min. Which action should the nurse take first? a. Document the finding in the chart. b. Initiate external pacing. c. Assess the clients medications. d. Administer 1 mg of atropine.

C Pacemaker cells in the conduction system decrease in number as a person ages, resulting in bradycardia. The nurse should check the medication reconciliation for medications that might cause such a drop in heart rate, then should inform the health care provider. Documentation is important, but it is not the priority action. The heart rate is not low enough for atropine or an external pacemaker to be needed.

3. A nurse assesses clients on a medical-surgical unit. Which client should the nurse identify as having the greatest risk for cardiovascular disease? a. An 86-year-old man with a history of asthma b. A 32-year-old Asian-American man with colorectal cancer c. A 45-year-old American Indian woman with diabetes mellitus d. A 53-year-old postmenopausal woman who is on hormone therapy

C The incidence of coronary artery disease and hypertension is higher in American Indians than in whites or Asian Americans. Diabetes mellitus increases the risk for hypertension and coronary artery disease in people o any race or ethnicity. Asthma, colorectal cancer, and hormone therapy do not increase risk for cardiovascular disease.

18. A nurse prepares a client for coronary artery bypass graft surgery. The client states, I am afraid I might die. How should the nurse respond? a. This is a routine surgery and the risk of death is very low. b. Would you like to speak with a chaplain prior to surgery? c. Tell me more about your concerns about the surgery. d. What support systems do you have to assist you?

C The nurse should discuss the clients feelings and concerns related to the surgery. The nurse should not provide false hope or push the clients concerns off on the chaplain. The nurse should address support systems after addressing the clients current issue.

To validate that a client has had a myocardial infarction (MI), the nurse assesses for positive findings on which tests? Creatine kinase-MB fraction (CK-MB) and alkaline phosphatase Homocysteine and C-reactive protein Total cholesterol, low-density lipoprotein cholesterol, and high-density lipoprotein cholesterol CK-MB and troponin

CK-MB and troponin CK-MB and troponin are the cardiac markers used to determine whether MI has occurred. Alkaline phosphatase is often elevated in liver disease. Homocysteine and C-reactive protein are markers of inflammation, which may represent risk for MI, but they are not diagnostic for MI. Elevated cholesterol levels are risks for MI, but they do not validate that an MI has occurred.

A client has just returned from coronary artery bypass graft surgery. For which finding does the nurse contact the surgeon? Temperature 98.2° F Chest tube drainage 175 mL last hour Serum potassium 3.9 mEq/L Incisional pain 6 on a scale of 0 to 10

Chest tube drainage 175 mL last hour Some bleeding is expected after surgery; however, the nurse should report chest drainage over 150 mL/hr to the surgeon. Although hypothermia is a common problem after surgery, a temperature of 98.2° F is a normal finding. Serum potassium of 3.9 mEq/L is a normal finding. Incisional pain of 6 on a scale of 0 to 10 is expected immediately after major surgery; the nurse should administer prescribed analgesics.

The nurse in the coronary care unit is caring for a group of clients who have had myocardial infarction. Which client does the nurse see first? Client with dyspnea on exertion when ambulating to the bathroom Client with third-degree heart block on the monitor Client with normal sinus rhythm and PR interval of 0.28 second Client who refuses to take heparin or nitroglycerin

Client with third-degree heart block on the monitor Third-degree heart block is a serious complication that indicates that a large portion of the left ventricle and conduction system are involved, so the client with the third-degree heart block should be seen first. Third-degree heart block usually requires pacemaker insertion. A normal rhythm with prolonged PR interval indicates first-degree heart block, which usually does not require treatment. The client with dyspnea on exertion when ambulating to the bathroom is not at immediate risk. The client's uncooperative behavior when refusing to take heparin or nitroglycerin may indicate fear or denial; he should be seen after emergency situations have been handled.

The nurse is concerned that a client who had myocardial infarction (MI) has developed cardiogenic shock. Which findings indicate shock? (Select all that apply.) Bradycardia Cool, diaphoretic skin Crackles in the lung fields Respiratory rate of 12 breaths/min Anxiety and restlessness Temperature of 100.4° F

Cool, diaphoretic skin Crackles in the lung fields Anxiety and restlessness The client with shock has cool, moist skin. Because of extensive tissue necrosis, the left ventricle cannot forward blood adequately, resulting in pulmonary congestion and crackles. Because of poor tissue perfusion, a change in mental status, anxiety, and restlessness are expected. All types of shock (except neurogenic) present with tachycardia, not bradycardia. Due to pulmonary congestion, a client with cardiogenic shock typically has tachypnea. Cardiogenic shock does not present with low-grade fever; this would be more likely to occur in pericarditis.

The client in the cardiac care unit has had a large myocardial infarction. How does the nurse recognize onset of left ventricular failure? Urine output of 1500 mL on the preceding day Crackles in the lung fields Pedal edema Expectoration of yellow sputum

Crackles in the lung fields Manifestations of left ventricular failure and pulmonary edema are noted by listening for crackles and identifying their locations in the lung fields. A urine output of 1500 mL is normal. Edema is a sign of right ventricular heart failure. Yellow sputum indicates the presence of white blood cells and possible infection.

19. An emergency department nurse triages clients who present with chest discomfort. Which client should the nurse plan to assess first? a. A 42-year-old female who describes her pain as a dull ache with numbness in her fingers b. A 49-year-old male who reports moderate pain that is worse on inspiration c. A 53-year-old female who reports substernal pain that radiates to her abdomen d. A 58-year-old male who describes his pain as intense stabbing that spreads across his chest

D All clients who have chest pain should be assessed more thoroughly. To determine which client should be seen first, the nurse must understand common differences in pain descriptions. Intense stabbing, vise-like substernal pain that spreads through the clients chest, arms, jaw, back, or neck is indicative of a myocardial infarction. The nurse should plan to see this client first to prevent cardiac cell death. A dull ache with numbness in the fingers is consistent with anxiety. Pain that gets worse with inspiration is usually related to a pleuropulmonary problem. Pain that spreads to the abdomen is often associated with an esophageal-gastric problem, especially when this pain is experienced by a male client. Female clients may experience abdominal discomfort with a myocardial event. Although clients with anxiety, pleuropulmonary, and esophageal-gastric problems should be seen, they are not a higher priority than myocardial infarction.

10. A nurse assesses a client who is scheduled for a cardiac catheterization. Which assessment should the nurse complete prior to this procedure? a. Clients level of anxiety b. Ability to turn self in bed c. Cardiac rhythm and heart rate d. Allergies to iodine-based agents

D Before the procedure, the nurse should ascertain whether the client has an allergy to iodine-containing preparations, such as seafood or local anesthetics. The contrast medium used during the procedure is iodine based. This allergy can cause a life-threatening reaction, so it is a high priority. Second, it is important for the nurse to assess anxiety, mobility, and baseline cardiac status.

2. A nurse assesses a client after administering a prescribed beta blocker. Which assessment should the nurse expect to find? a. Blood pressure increased from 98/42 mm Hg to 132/60 mm Hg b. Respiratory rate decreased from 25 breaths/min to 14 breaths/min c. Oxygen saturation increased from 88% to 96% d. Pulse decreased from 100 beats/min to 80 beats/min

D Beta blockers block the stimulation of beta1-adrenergic receptors. They block the sympathetic (fight-or-flight) response and decrease the heart rate (HR). The beta blocker will decrease HR and blood pressure, increasing ventricular filling time. It usually does not have effects on beta2-adrenergic receptor sites. Cardiac output will drop because of decreased HR.

15. A nurse cares for a client who has advanced cardiac disease and states, I am having trouble sleeping at night. How should the nurse respond? a. I will consult the provider to prescribe a sleep study to determine the problem. b. You become hypoxic while sleeping; oxygen therapy via nasal cannula will help. c. A continuous positive airway pressure, or CPAP, breathing mask will help you breathe at night. d. Use pillows to elevate your head and chest while you are sleeping.

D The client is experiencing orthopnea (shortness of breath while lying flat). The nurse should teach the client to elevate the head and chest with pillows or sleep in a recliner. A sleep study is not necessary to diagnose this client. Oxygen and CPAP will not help a client with orthopnea.


Related study sets

Macroeconomics Exam 3 Chapter 20

View Set

ECO 361: Chapter 5, ECO 361: Chapter 6, ECO 361: Chapter 7, ECO 361: Chapter 8, Econ 335 final exam Ch. 7, Econ 335 final exam ch.10, Econ 335 final exam ch. 11, Econ 335 final exam ch. 12, Econ 335 final exam ch.15, ECON CH. 16, Pre Exam 1, ECON3171...

View Set

Ch 7: Brokerage Relationships: Law & Practice

View Set